Comprehensive coverage

Dark matter throws a clue / Clara Moskowitz

After decades of negative findings, physicists have finally found a clue

Colliding galaxies in the cluster Obel 3827. Courtesy of the European Southern Observatory (ESO), NASA and the European Space Agency/Hubble Space Telescope
Colliding galaxies in the cluster Obel 3827. Courtesy of the European Southern Observatory (ESO), NASA and the European Space Agency/Hubble Space Telescope

There is something somewhere in the universe. We can't see it, we can't touch it and we only know it's there because of the gravitational pull it exerts on the objects in the universe. For decades, the story of dark matter has been discovery after discovery about what this mysterious substance is, a gradual filtering of possibilities that has made physicists feel increasingly nervous. What will happen when the last candidate is deleted from the list? Will our fate be sealed, never to get a glimpse of the nature of the matter that contributes about 25% of the total mass in the universe?

The blue spots show bent light, which astronomers use to measure four merging galaxies.

This grim scenario took a hopeful turn in the early spring of 2015. Researchers uncovered one of the most intriguing clues in years: a sign of the existence of a new force that may allow dark matter to "talk" to itself. This insight will help explain what kind of particles, perhaps, dark matter is made of.

This clue came from observations of a corner of the universe known as Obel 3827. Astronomers recently pinpointed the location of dark matter within four colliding galaxies in this cluster by using a phenomenon known as gravitational bending (the bending of light as it passes near massive objects). Observations made with the Hubble Space Telescope and the "Very Large Telescope" in Chile have revealed that the dark matter surrounding at least one of these galaxies is trailing behind the normal matter there. The observation implies that the dark matter particles interact with each other and slow themselves down: a phenomenon that has never been seen before.

A team of astronomers led by Richard Massey of the University of Durham in England speculates that since the interactions did not affect normal matter, they must have occurred through a force other than gravity, a force that only affects dark matter. The exchange of "dark photons", for example, may create this force. Such a situation may be analogous to the way in which ordinary protons interact with each other through the electromagnetic force: when two protons approach each other, each of them releases a photon, the carrier of the force of electromagnetism, and the other absorbs it. These exchanges transfer momentum, causing the two protons to separate from each other.

This news excited the physicists and inspired them to look for answers. "If this discovery holds up, then it is much more than sensational news," says physicist Neil Weiner of New York University, who did not take part in the research. A scenario involving dark photons is a modification of the most basic and accepted concept of dark matter as consisting of one and only one type of particle, which is commonly called a "weakly interacting massive particle", or WIMP ("weak", in English). But the idea that dark matter is accompanied by dark photons and exotic interactions may help solve some problems raised by describing dark matter using one type of WIMP, such as why the centers of galaxies are less dense than expected.

This concept may also help physicists narrow down the list of dark matter candidates to a great extent. “Although we have evidence for the existence of dark matter from a huge variety of sources,” says Weiner, “at present we have nothing that clearly preserves anything beyond its gravitational interactions. If it turns out that it has self-interactions at this level, we can eliminate a huge number of models" that describe what dark matter might be. In particular, this finding, published online in April and in print in June 2015 in the journal Monthly Notices of the Royal Astronomical Society, may contradict many popular versions of the hypothesis that dark matter is a particle predicted by the theory of supersymmetry. This theory, a tempting idea that tries to explain many mysteries in physics, such as why the mass of the Higgs boson is so low, assumes that there are other particles in the universe that have not yet been discovered. However, if one of these particles (which may be a WIMP) is responsible for dark matter, then most versions of the theory do not predict self-interactions.

The paper's other authors say it is too early to rule out a more everyday explanation for their observations. For example, it is possible that dark matter located outside the colliding galaxies, but within the line of sight from Earth, may contribute to the gravitational pull. "One of the caveats of this new study is that it's only one bone," says team member David Harvey of the Swiss Institute of Technology in Lausanne. "We don't know what we don't know, and these unknown factors may affect the outcome." In addition, previous searches in other clusters did not reveal signs of dark matter interacting with itself, including a study published in March 2015 in the journal Science, headed by Harvey, who analyzed 72 collisions of galaxy clusters, rather than individual galaxies. However, clusters collide faster than galaxies, so there is less time for dark matter to interact and lag behind, so the two findings are not mutually exclusive.

If it turns out that the latest observations do not reflect new forces or interactions of dark matter, Obel 3827 will become another example of what dark matter is not. Meanwhile, the search for its particles in underground detectors continues to turn up clay. Additionally, dark matter has yet to appear at CERN's Large Hadron Collider (LHC). The scientists hope that these trends will change soon: the accelerator resumed operation in April 2015, at the highest energies it has ever reached, and the detectors are now incredibly sensitive. "Dark matter has been so elusive until now, but we've never had the data that's coming our way," says Harvey. "In my opinion, it's now or never."

235 תגובות

  1. Israel Shapira
    Believe me, I read several books on the subject. I understood them. Not you. It is possible to understand a Gemara page you are not. You have an extraordinary talent to take a topic that is enough to write a paragraph about, two at the most and you. Take a pen and empty all the ink. Richard Feynman who received a Nobel Prize would not understand you. It is possible to write about a subject in the world as complex as it may be in a concise and matter-of-fact manner. You don't know how to do that. Too bad. It seems that the spirit of postmodernism is upon you. With this I end my argument with you.

  2. You have no pretensions..

    And what pretensions do you think we have? How did you deduce that? That we ask questions that you don't even understand?

    Come on, back to the cage.

  3. Eric and Israel Shapira
    I have no pretensions, you do and this is how you look. Walking around in the zero waters of photons and quarks and thinking you understand. You're wrong. The Huygens bosons will still drive your mind crazy, even so it is not sure that it is clear. You drank a lot of vodka.

  4. Well Gorilon, what scientific breakthrough did you reach? Peeling a banana using the tail? And what award did you get? The Coconut Cup?

    Maybe you can explain to us how the earth attracts the moon? With Tarzan's rope?

    Humanity holds its breath.

    Also the jungle animals.

    forward!

  5. Dear gorilla
    We are all human apes trying to understand things and exchange experiences. After we all present our explanations in turn, we understand the material better. And from all my teaching I am educated!
    good week
    A little rose from the tree
    Yehuda

  6. To all the responders of all kinds. Maybe stop playing the role of philosophers on a dime. None of you came up with some kind of scientific breakthrough. You have not received and will not receive a Nobel Prize. You move in circles like a dog chasing its tail. Maybe do something more productive like reading a book. As they say, boredom is the mother of all sins. From the height of my years I pity you.

  7. blow up an egg, which god do you mean?
    To Buddha? To Zeus? to Odin? Maybe for Allah?
    During the written human history, people believed in about 1000 different gods. Choose one and I'll be happy if you explain your choice.

  8. elbentzo,
    Please accept my apologies.
    A few years ago I tried to share my thoughts here on the science website by way of comments and conversations with friends. I started from a starting point of a "universe from nothing" and tried to show that "nothing" is an entity (more precisely, nothingness) with one active property - negation. I tried to explain how something was created from nothing without external intervention (of the kind of God that Tambaltso mentioned). I even enlisted the help of puns (no-no gives yes, but yes-yes does not give no). However, I always encountered a wall of misunderstanding of the type "but where do you harness the horse", such as "what is the mass of a particle that creates mass?". I made a claim that as the "imaginary" number i is the necessity of reality, there is also a logical truth value whose application to itself gives FALSE; We do not know it in everyday reality, but its existence solves the problem of asymmetry in logic and shows that the two known truth values, TRUE & FALSE, derive from it, as well as a solution to the question of why the space we operate in is three-dimensional.
    For a moment I thought you had already visited here...

  9. jubilee
    The only thing that doesn't need to be explained in this world is God.
    He has already explained himself well.
    We just have to try to understand what he meant.
    : )

  10. jubilee,

    1. I'm not kidding you.

    2. I have no idea who you are and have never read anything you've written (unless you posted it on JHEP or something, although I think I'd remember your name then).

    3. You are not the only one who wanted to define an abstract mathematical structure from whose mathematical properties alone all physical phenomena would arise. They did it before you. In fact, Torah Sdot is exactly such a Torah. There is no particle in it, no definition of mass or force. All there is is a vector space (Fock Space), on which are defined a series of copies with certain symmetry properties, a vacuum (in the mathematical sense, not in the sense of empty), etc. From these definitions we *get* the particles, we get the concept of forces and persistence, we get what mass is, etc.

  11. Tambaltso,
    If you want to bring God into the picture, that's your business.
    But please keep in mind that he also needs to be explained. So what did we gain?

  12. At least to me, it sounds like Hamlet.
    And I'm not talking about the play.
    I'm talking about the food made from eggs.
    Why don't we go further?
    - God invented and determined the particles?

  13. elbentzo
    Are you kidding me. Right?
    I don't know who is hiding behind your nickname (I can only guess), but I suspect you got to read chapters from my model 🙂
    Indeed, I start with a mathematical structure, lacking any physical properties, and through mathematical analysis alone I arrive at complex structures that I attribute to dark matter particles.

  14. Due to the mass-related decay phenomenon (proved by Eddington's observation), the physicists assumed that electromagnetic waves in the particles (photons) are endowed with mass. Since no such mass was measured, they invented the concept of "movement mass".
    The aging phenomenon is just one of several phenomena linked to mass. Two other well known ones are gravitation and persistence. These are three essentially different properties, and I am interested in knowing what is the mechanism that produces them and other possible phenomena.

  15. jubilee,

    Why don't we go one step further? Leave you particles. Let's define only a mathematical structure, devoid of any physical properties, which according to its structure (mainly the symmetries but not only) will determine which particles there are. That is, you can look at physics where particles are not elementary things at all. How does that sound to you?

  16. Yoda, Israel
    The Higgs boson has mass as a result of interactions within the Higgs field (if I understand correctly).
    And don't forget that there is such a thing as "dark energy". Maybe it "gives" mass to dark matter particles? Or gravity pushing particles?
    Ugh... I'm already getting pressure differences in my brain..

  17. Yoda

    I'm not smarter than you, but I did ask this question. According to "Breaking symmetry" Elek.

    But in a simple mechanical model like pushing, I don't see how the particle that gives mass to bodies would have mass itself.

    Understand?

  18. To Israel
    I don't understand, and I will ask the question that surely these are already smarter than me: if the Higgs boson gives the mass to someone who has no mass then who gives the mass to the Higgs boson?? The Higgs boson itself?…. So if he can, why can't everyone give themselves the mass?? . We may come across a well-known paradox here, the "paradox of the book" which says: if the book tells everyone who doesn't cut their own hair, then who tells the book???
    Food for thought.
    Come on today there is an interesting lecture at the association in Givatayim: nuclear solar. need to prepare.
    good evening
    Sabdarmish Yehuda

  19. Israel and Judah (United Kingdom) 🙂

    Israel: Mesa is also expressed in Idus

    Yehuda: I have no problem with gravity pushing particles. It is not impossible that they do fulfill and carry out everything you impose on them. They have mass, they have momentum, kinetic energy and everything needed to produce physics. They are just not thorough enough for my taste. I'm looking for a being that is devoid of all those basic physical attributes. Because, otherwise, we have not progressed at all.
    Azrom with you: Let's build a model based on a particle with no physical properties, from whose behavior - as an individual or as a group - gravitational pushing particles are created.

  20. Yes, mass manifests itself in two features:

    1. Resistance to acceleration (inertia).

    2. Gravitational attraction.

    In all the measurements conducted so far, there is no measurable difference between these 2 properties of mass, so we can see them as one property.

    In gravitational pushing, without the particles there is no gravitational mass. The earth and the sun can be a meter apart without any attraction between them. Therefore, from what was said in the previous section, there is no inertial mass without the particles either.

    Therefore, if bodies have no mass without the particles, how is it possible for the particles themselves to have mass? What smaller particles give them their valve? Higgs light bosons?

    ?

    ??

    ??!?

  21. I don't understand what a particle without mass is, as far as I understand the concept of a particle if and only if it has mass. True, I know that for example a photon has no rest mass and some other strange definitions, but I don't have to apologize because my particle has mass. And if I define that it has momentum, will that satisfy you? What about kinetic energy, is it allowed to have it? My particle has everything needed for gravitational pushing, and if there is a Higgs boson, there is a problem with that, then Zebsho (it's his problem)
    Please respond gently
    Good Day
    Yehuda

  22. Yes, if the pushing gravity particles have mass and are also the source of mass for all baryonic matter, then what gives mass to the pushing gravity particles? Smaller particles? Which in turn get the valve from even smaller particles? In turn... Partinconocones... Partinconconones... In turn..

  23. Yehuda,
    I have no mass. If we define mass by means of mass, then we have not gained anything.
    Mass is an expression of a phenomenon caused by the behavior of a collection of dark matter particles.

  24. Interesting Yuval, also in Pushing Gravity many claim that the particles of Pushing Gravity make up the baryonic matter and with you it's the same except that they are dark maybe in the end it's the same lady in a change of glory??, food for thought.
    The emphasis should be what you say: "to first define the spatial and mathematical properties of the dark matter particle." ” End quote. Or in other words…. Define the particle of gravity pushing. For me, its mass is 6.3 times ten to the power of minus 37 grams, what about you??
    Good night
    Yehuda

  25. Thank you Avi
    The principle behind the model that I came up with 40 years ago and that today I attribute to dark matter is based on the principle of simplicity.
    Physics today represents a large "zoo" of particles and forces which on the one hand seem different from each other but on the other hand "talk" to each other. Most of them share certain characteristics, the first of which is the mass characteristic. One division I make in relation to this property, mass, is those in which it was measured and those in which it was not measured or was measured only indirectly.
    The universe is populated by dark matter particles with a density that varies from place to place. At high density they form structures we know as baryonic matter. At low density they create what we call empty space. In baryonic matter we can measure mass. The propagation of electromagnetic waves is possible in empty space.
    To answer the question of how the dark matter particles arrange themselves to form baryonic structures on the one hand and empty space on the other and how the laws of physics are constructed in general, the spatial and mathematical properties of the dark matter particle must first be defined. But one must be careful not to give it any known physical properties, since it is supposed to build the known physics and not be built from it.

  26. Thanks Ariel. I changed the website as you suggested and I am waiting patiently to see what will happen.

    Thank you too, Yehuda. Without going into the details of your model, I'll just say that I don't like the pressure differences you're talking about. This, for the simple reason that pressure is a physical phenomenon defined as a function of a more basic physical quantity (force), and it is also defined as a function of a more basic physical quantity (mass); And in this way a circular definition is obtained that does not lead us anywhere.

  27. Raphael
    Good question! When we look at another galaxy, like Andromeda, the size of the galaxy is small compared to the distance. Therefore, it can be assumed that all the light from the galaxy arrives at the same time.

    In addition, time is small relative to the rate of change of the relative position of the stars. So, if we look at our galaxy, an error of tens of thousands of years does not cause a fundamental error in the position of the stars.

  28. to Ariel
    It doesn't have to be the exponent of the hold these could be endless other possibilities. For example, the gravitational constant G may be slightly different (infinite possibilities), etc., at close distances we checked and found that Newton's formula with the 2nd power of the distance is suitable for us, but it is only up to a thousand light years beyond that, any attempt to use the formula shows that there are creaks in the formula, for example Pioneer's anomaly , Dark Mass, and more.
    To Yuval Chaikin
    You said: "There is still not much knowledge about the properties of dark matter, but there is certainty about its existence. This, based on two observed phenomena: gravitational dusting and an anomaly in the movement of stars at the edges of galaxies." End quote.
    You said that we would observe a gravitational cloud, and you are wrong because what you observed is only cloud, gravitational or not is only your explanation. There are many other types of clouding, for example clouding created by a lens or a pressure difference... As for the anomaly in the movement of stars at the edge of the galaxy, then the debate is going on here. What is it and why is it happening?
    Good evening here in Israel
    Yehuda

  29. Yuval I think changing the link will solve the problem. Change to the scientist's home page and your comments will not have to wait for release (the reason is to prevent publication)

  30. I have a question that has been on my mind for a long time. Maybe everyone already knows the answer except me. When we look at the universe, the picture we see is not a snapshot of one particular point in time, each point belongs to another time. For example when we look at a galaxy then there are stars that are closer to us and there are those that are far from those stars that are close to tens of light years or more. How can it be determined in such a way that the galaxy is indeed in the shape of a spiral or any other shape that appears to us?

  31. Yehuda, you still haven't explained the meaning of large distances... what are you claiming? that the greater the distance, the smaller the exponent will be? (the exponent will be given as a function of the distance)
    Or do you mean that the exponent is constant but it is different from 2?

  32. Electrophysical power powder, let it be…
    I also bring this response in the parallel universe, since there is no need to wait for release. It won't hurt you to open a Facebook account. It's free.

    "Great distances"? A distance is a distance of any size and there is no reason to state that at certain distances different physics will operate than at other distances.

    There is still not much knowledge about the properties of dark matter, but there is certainty about its existence. This, based on two observed phenomena: gravitational dusting and an anomaly in the movement of stars at the edges of galaxies.

    How do the body and gravity form out of the dark matter or why do I claim that a body does not produce gravity? Well, the property mass manifests itself in at least two different and independent ways, which general relativity claims are two aspects of the same property - persistence and attraction - and also successfully predicts the phenomenon of gravitational acceleration. The existence of two different modes of the same feature requires the search for the mechanism that causes both and also the additional phenomenon that was predicted (and confirmed by Eddington).
    According to the model developed by Newton, the force of attraction between two bodies is proportional to the product of their masses and inversely proportional to the square of the distance between them. In the absence of additional knowledge, it is easy to jump to the conclusion that it is the masses of the bodies that create the force of attraction between them, and this is indeed what physicists do to this day. The phenomenon of persistence is also characteristic of the mass, as is the phenomenon of obsolescence. Because of the similarity to other physical phenomena, it was hypothesized that gravitation is a dual phenomenon - wave/particle - that is carried across an unknown medium by means of particles called "gravitons", but those particles have not been recruited, at the moment, to also explain the phenomenon of persistence and decay. As of today, gravitons have not yet been discovered and the mechanism that produces the various phenomena attributed to mass has not yet been discovered. The discovery of dark matter gives hope that the long-awaited discovery will come soon.

    Not only you play poker on Tuesdays and not only Yehuda lives on Eli Cohen Street in Herzliya.

  33. For Mr. Power powder
    I will try to explain to you what is meant by large distances
    In Newton's gravitation formula there is an R factor that expresses the distance between the two gravitationally attracted bodies. Small distances are the inner solar system up to 60 astronomical units, which is about ten billion km (about a thousand light years)
    Newton's gravitation formula calculates for this distance data that is almost identical to reality. Small even if it is two stars in Andromeda.
    But let's assume that R is 60,000 astronomical units - about 10,000 billion km which is about one light year, can we even now be sure that Newton's formula will give correct predictions?. And it is still considered a distance
    And what about R, which is the distance from the center of our galaxy to the center of the Andromeda galaxy, about two million light-years, which is 20 million million million km, it is already a long distance. Even then, will we be sure that Newton's formula will be correct for this distance? I mean, will it even then show us the exact gravitational force between the Milky Way and Andromeda?, and what about distances of billions of light years????
    Got it Mr. E-Powder?

    Good Day
    Sabdarmish Yehuda

  34. Israel, Yuval, Yoda
    Thanks. But I still did not understand the meaning of the saying: "at great distances"?

    Arkman
    great hairstyle 🙂

    So you claim - like the standard model - that gravity is a separate force. Stands on its own, like the weak force for example. Did I understand you right?
    Second, how do the body and gravity form out of dark matter? The accuracy of the particles? 🙂 I would appreciate it if you could explain.
    After all, there is still no knowledge about the properties of dark matter, what's more, even certainty about its existence is still very much in doubt. All that is known is that there is contamination and that it is - probably - due to some other substance. Dark.

    Another thing: why do you claim that a body does not produce gravitation? Doesn't the mass of a body create some force that attracts a body of lower mass?
    Maybe it's just me, but I always thought that bodies create a force called gravity, but we still don't know what the particles that make up this force are.
    Well, well... Tuesday. poker.
    I will go to my garden to pick some 80-year-old dark tomatoes and send them to Eli Cohen Street in Herzliya anonymously, while hoping that I will receive back 80 grams of Turkish Delight.
    By the way, Yuval, I would appreciate it if you would respond here because I am not a Facebook or Twitter subscriber and all that.

  35. albentezo,
    Following on from our conversation, the following quotes from Carol's blog come to explain why I wonder what a *spectator* is (and for others who might be interested)
    they're due to an effective measurement of the quantum state of the inflaton field when the universe reheats at the end of inflation

    But when we're not looking, a system in its ground state (like an electron in its lowest-energy orbital around an atomic nucleus) isn't fluctuating at all; it's just sitting there

    but there are no particle detectors around, so in fact the quantum fields are sitting there quietly in a stationary state with no definite particle number. Therefore, these kinds of fluctuations aren't "really happening."

    Then I read this:
    the microscopic states of the plasma provide an environment that becomes entangled with the large-scale fluctuations of the inflaton, effectively measuring it and collapsing the wave function

    Does this mean that every classical "element" is actually an observer? Can I conclude that anything that is not classical cannot be an observer that will cause the wave function to collapse?

    I would appreciate it if you would revisit the idea of ​​a universe from nothing, since, if I understand correctly, Carroll is indeed talking about pertubations, but he claims that they occurred at the end of the inflation phase (and if I understand correctly, this is a situation in which there are already classical structures that serve as observers) but in an inflating universe that only has Vacuum energy and no scouts, no quantum fluctuations at all (this is what he wrote. I added the topic of scouts). Why would it be different in the state of a cold universe (hit death)
    I understand that he is basing himself on multiple universes but under this assumption, if he and his colleagues are right, is there still a problem here with the idea of ​​a universe from nothing?

  36. Yehuda,
    The monster hurricane forms where most of the winds are concentrated and where most of the mass is. In the Bolt Cluster, gravity is not where most of the particles we know are. How do you explain that?
    Why do you oppose dark matter (which you don't know about at all) by introducing another, super problematic dark matter that is easy to disprove with a simple experiment as Albentzen suggested? How can you not see it?

  37. Yehuda
    This does not explain the distortion in the spirals that is already created at large distances. Only attraction can explain it.

  38. Shlomi Narkis is a Swedish name. To Tomi I thought that the name Yuval Chaikin lights up a red light. I will further unravel the mystery of selective blocking. In any case, I also respond in the corresponding article on Facebook which is the link to it
    https://www.facebook.com/hayadan/posts/1026656190686882

    Shmulik,
    The dark matter particles and the spaces between them are what make up the matter on the one hand, and constitute the medium in which the electromagnetic waves propagate on the other hand. The quantitative ratio between the particles and the spaces (density) is what determines the size of the material structures and the transmission speed of the electromagnetic waves. A massive body is formed from a high density of dark matter particles. A cascade of concentrations also results in high density in the vicinity of the massive body, and this is what slows down the movement of electromagnetic waves and results in the bending of light.

    Alchemy has already gone out of fashion and the dark matter model that I bring is compatible with the second law of thermodynamics.
    In the end it won't be very interesting.

  39. Yehuda

    You write "the particles themselves move around the sun like any other matter".

    Nice, but in which direction? And at what speed?

    Is it possible that each body that orbits the sun has its own particles adjusted? And what about the moon that revolves around the earth? And the Apollo spacecraft orbiting the moon? And Daphne who turns Apollo and turns his head?..

    But do not lose heart, our brother. Spend a pleasant time with the children and grandchildren.

    There is an elegant solution to the friction problem, but that doesn't mean that gravity pushing is a real explanation.

  40. Miracles
    Want an explanation other than gravity?,,,, please
    Two hurricanes moved across the ocean towards each other and collided and a bigger hurricane was created - a real case that I remember personal information. Gravity didn't work here. Differential pressure? Yes!
    It seems to me that this is also possible for galaxies,
    So this is me jumping to the grandchildren
    Bye
    Yehuda

  41. Yehuda
    We know from observations that galaxies collide, and we also see in photographs the effect of gravity on the shape of galaxies.

    I would love to hear another explanation for these photographs.

  42. Shmulik and Ariel
    It could be that you are right and the dark matter exists and with it the energy flew to her and I also explained the matter of the speed of light ten times so let's just say goodbye as friends I am tired for almost a week just reacting and standing under fire and darkness and I have to finish a job for a client who is waiting for her tomorrow and my lovely grandchildren have also arrived. So it was nice and even pleasant
    All the best bye and bye and please don't get angry.
    Yehuda

  43. Ariel,

    You claim not to be a physicist, but maybe you should have been. In short, you made a note about Yehuda's mistake (well, one of Yehuda's many, very, very many mistakes) and also about how to measure it and show him in an experiment that he is wrong (if he will listen to you is another question, and my guess is of course - no).

    Since I am only repeating what you have already said, I will summarize in two lines - what Yehuda describes is the expansion of matter within infinite space, not the expansion of space. These are completely different things. The way to differentiate between them is to check if we feel acceleration. An accelerating body knows that it is accelerating (it is a measurable quantity), and we know by definition that we are *not* accelerating out of some center of the universe (in Yehuda's description there must be a center for the gas cluster, i.e. for the distribution of matter in the universe, and we know that this is not the case), but that the universe is expanding throughout point isotropically and that this expansion becomes faster. This is not acceleration in the sense of an accelerating body (on which a force acts), but a space in which all distances are lengthening at an increasing rate.

    But despite Ariel's beautiful distinction, there is something more basic that can be measured in a high school laboratory, and this is what I wrote in the previous comment. All that needs to be done to disprove the mechanical Torah (and indeed this is the first way it was disproved, although there is a huge collection of refutations) is to throw a ball in a room with no air resistance and check if friction is mysteriously created out of nowhere.

  44. Yehuda,
    I don't care what you use, except that the theory of relativity successfully predicts so many things including the prohibition of matter with mass to cross the speed of light. Therefore, you do not see the problem of your proposal?

    You didn't answer how your idea explains the prominent cluster???

    You didn't answer how you don't see the categorical contradiction in promoting dark matter while shouting from here until a new announcement that there is no dark matter.
    By definition, you're talking about particles we haven't discovered yet. It's h u m r a p l !!!!

  45. You have some mistakes that are probably due to a lack of understanding.
    1) The expansion you describe is the expansion of matter not of space and that is the expansion of the universe.
    2) A star that is at rest will not feel acceleration, it will look as if the other stars are moving away from it and the reason is again, the expansion is of the space between the stars, meaning a new space is created between them.
    3) The acceleration of the universe is on the rise. She doesn't play yo-yo. There is something called Hubble's constant and it says that for every million persecs (persecs = 3.26 light years) the speed of the object relative to us will appear greater by another 69.32 km per second.
    That is, the greater the distance, the greater the speed.
    4) Eventually your expansion will reach a state of equilibrium with gravity (whether you are right about the distance exponent or not) and a static universe will be created.

  46. Ariel
    My pressure difference nicely explains the acceleration in the expansion of the universe without the need for dark energy. So simple that you want to cry!
    So please sharpen your senses because I will not explain twice. Well, let's conduct an experiment:-
    Into a huge vacuum container with an infinite diameter, insert a gaseous sphere with a diameter of 100 light years, inside which stars are scattered. Initially layer number one hundred will spread into the void - the outermost layer number 99 will not move at all because the information about the void will arrive only after a year and cannot move faster than the speed of light. After a year, the innermost layer 99 will also begin to move, and after another year, layer number 98, and after ten years, layer number 90. So far, do you agree, Ariel? If you agree, then you agree that a star in an internal declination does not move, but after some years it will start to move outwards with increasing speed - and this means accelerated movement. No need for dark energy and plaster!
    And not only that, it also explains the nature of acceleration at first it will increase and then decrease but there will always be acceleration.
    So here, everyone says there is dark energy and even a lot, and I, Yehuda Sabdarmish, say there is nothing.
    So in these days of falling prices in all the stock exchanges in the world, I would not invest in dark mass or energy.
    The dark evening descends then
    Bye everyone
    Yehuda

  47. Yehuda Admittedly I don't know if only mass/dark energy is the only solution and most likely not but I'm not a physicist so I'll wait with that. Your pressure difference simply cannot explain the acceleration in the rate of expansion of the universe for the simple reason:
    The pressure of a gas depends on several factors: the density of the gas (mass/volume) temperature (the ideal gas equation).
    Now according to your claim, the pressure difference explains the acceleration in the rate of expansion of the universe, but as the universe grows, the volume increases, the universe cools, and the pressure of your gases decreases, which should show a decrease in the acceleration of the expansion, which does not agree with the results of the experiments, which show that the rate of expansion depends on the distance (dark energy explains this).

  48. Floating point and floating point
    It seems to me that Le Sage's Gravity Pushing is different from my Gravity Pushing. My simple, gaseous universe talks about a small particle whose size is less than ten to the power of minus 36 grams and whose speed is only the speed of light. Regarding the precious speed of light. I did a Newtonian calculation and got 1.3 times the speed of light. And I know that the speed of light is maximum, so probably the relativistic speed should be a little less than the speed of light, but am I allowed to use the theory of relativity when I'm testing a foreign idea? It's like I'll use your car just because they'll think we're friends. They could rightly argue that I should not use the theory of relativity. So I left it open and everyone will decide what suits them.
    Miracles
    Where did you get this statement? What I am claiming based on my gravitational pushing which also includes a gaseous universe is that what appears to be our certain collision with Andromeda will not necessarily be so. The question is whether the universe is gaseous or not. And by the way, if you've already mentioned it, then it's possible that a galaxy that contains a lot of dark matter was rotating in dark matter, it will also have a Magnus effect!!!. We will wait 4 billion years, we will see and decide!
    Good Day
    Yehuda

  49. Yehuda,

    I already showed you years ago a list of problems that have no explanation without dark matter. I even gave you a number of links to scientific articles concerning positive evidence for the existence of dark matter. You always refused to read them.

    Want one more time? you are welcome. Show me how without dark matter or dark energy you explain the power spectrum of the cosmic background radiation (in particular the structure of its ups and downs).

  50. Yehuda,

    You seem to insist on not understanding the very simple issue of friction. In one of the last comments you said that an experiment might be needed. So here, I have an experiment for you.

    Let's take a room with a vacuum over KDA. In this room, fire a cannon ball parallel to the ground (below, X-axis). According to general relativity (and Newtonian gravity, which is a special case of the weak field approximation), no force will act on the x-axis. On the vee axis (radial from the center of the center), on the other hand, there will be an approximately constant force with an acceleration of about 10 meters per second squared. That is, this theory predicts a certain trajectory - a perfect parabolic trajectory.

    The mechanical theory of gravity also predicts a certain trajectory, but a different one. Why? In addition to the force of gravity it predicts (which in this case is the same as Newtonian gravity, or in your language - short distances), it also predicts friction in the X-axis against the direction of movement. A literal explanation for this is found in a very nice and simple way in Feynman's lecture that you linked to - the cannon ball moves in a certain direction and in the direction of its movement it collides with many more particles than in the opposite direction (because it chases them and does not run away from them, like in the rain). Therefore the pressure differential you love so much will push it back into the barrel.

    This is a simple experiment that can be done in any laboratory. In fact, when I was teaching labs as a master's student at a university somewhere-around-here, I ran a very similar experiment. Now I'll let you guess - when you do this experiment on the surface of the Earth, do you find that the cannon ball moves freely on the X-axis without forces, or is there a mysterious force of friction that slows its movement on this axis even though it is a vacuum?

  51. to Ariel
    If you show me a problem that can only be solved with the help of mass and dark energy, then I apologize to everyone and invite you to a fun meal. You have no idea what a burden you will take off my shoulders.
    Yehuda

  52. albentezo,
    thanks for the answers. I will find James Gates' lecture on the subject. She was a little more detailed. Regarding Carroll's language, I need to think a bit about what bothers me (if I understood correctly, he explains why Boltzmann brains will not be formed but allows the formation of an entire universe?)

  53. Yehuda,
    Beyond everything that all the others write, I must state that in my life I have never heard of a calculation that can yield two different speeds. What is this, a program as you wish? In my life I have never heard of such a strong belief in particles flying close to the speed of light on both sides, which we don't detect, while endlessly abhorring what physicists call dark matter. You really don't realize that you are replacing dark matter that the physicists claim can explain the bullet cluster with another dark matter that according to Vicky (who is, what to do, more reliable than you) should have a much higher speed than light, while yours is close to the speed of light on both sides (depending on how many keys on you) and that in any case, if it is faster than light then it completely contradicts relativity and cannot explain the bullet cluster because what is happening there is that most of the gravity is not where most of the known matter is. How can your particles explain this? Pressure difference really can't explain it. Pressure difference is just a mantra.
    Don't you realize that field theory and the standard model put an energetic limit on particles interacting with known matter and any new candidate cannot make the interaction you are asking your dark matter to make?

  54. Yehuda,
    Beyond everything that all the others write, I must state that in my life I have never heard of a calculation that can yield two different speeds. What is this, a program as you wish? In my life I have never heard of such a strong belief in particles flying close to the speed of light on both sides, which we don't detect, while endlessly abhorring what physicists call dark matter. You really don't realize that you are replacing dark matter that the physicists claim can explain the bullet cluster with another dark matter that according to Vicky (who is, what to do, more reliable than you) should have a much higher speed than light, while yours is close to the speed of light on both sides (depending on how many keys on you) and that in any case, if it is faster than light, then Zebhas completely contradicts relativity and cannot explain the bullet cluster because what happens there is that most of the gravity is not where most of the known matter is.
    Don't you realize that field theory and the standard model put an energetic limit on particles interacting with known matter and any new candidate cannot make the interaction you are asking your dark matter to make?

  55. Yehuda
    I didn't understand 🙂 You tell me - "Your observation does not fit my theory, so I choose to ignore it".

    Sounds a bit like religion to me...

  56. Yehuda, if I present to you a problem that you claimed your theory solves without the need for mass/dark energy, will you abandon your current theory?

  57. for miracles
    I'll start with Andromeda. Don't be so sure we'll bump into her. I don't have gravitation and the gaseous universe with the rotation of the galaxies will, as you know, produce a Magnus effect and the galaxies may disappoint Newton. Because what is the chance that two rotated footballs will meet?. negligible!.
    And to your first question. When Einstein wrote his theory he added a cosmological constant which he later apologized for and in the end he was right. What needs to be done is a tiny patch to fit my tiny patch. Look, even in circumcision sometimes corrections are made.
    The truth is I didn't take care of this "entertainment" - I meant: "Relativistic Gravity Pushing" (all rights reserved on the name) and maybe you should really see what the essence of the change that needs to be done is.
    Come on, another weekend is gone!
    Yehuda.

  58. Yehuda
    And one more little thing. You are saying, as I understand it, that the gravity formula is wrong at distances of tens, or hundreds, of light years.
    So, why are we going to collide with Andromeda, even before they finish the light rail in Tel Aviv?

  59. Yehuda
    General relativity expresses gravity using a "simple" formula that gives the same result as Newton's equation. I understand that Tatha suggests a modification of the mathematics instead of dark matter. It's hard for me to accept that because we didn't find particle tagging of that dark matter in accelerators, that Einstein's math is wrong.

    Newton was wrong, because his basic assumptions were not accurate. Which basic assumption of Einstein's is wrong, in your opinion?

  60. Thank you Israel for mentioning Dunedin. From now on I would like to use this endearing nickname instead of the gloomy term "dark".
    Denidine particles and the spaces between them are responsible for the various physics phenomena, and the differences between the various phenomena are determined by the ratio between the denidine and the spaces, i.e. the density.
    The reason why we have not found Dunedin in our immediate vicinity is precisely because it is invisible. And just as Shraga Gafni's Dunedin (Un Sharig) can be discovered by tricks, also to reveal our Dunedin we need tricks that I trust the human genius to find such.

  61. Say Yehuda, who is the talented genius who filmed your lecture on YouTube? The quality of the photography is simply horrible, starting from the third minute you can hardly see silhouettes, and also the quality of the sound right in your face.

    How does a person manage to reach such a high-quality result?

    At least you scolded him?

  62. to a floating point and to Israel
    Friction has existed since the formation of the solar system and it exists even today even without agreeing with the pushing gravity, and in the solar system and the planets have overcome it, at least some of the planets. Anyone moving in a suitable orbit of space and speed continued to revolve around the sun. You raised a problem of direction so that there must be a vector of force in the direction of motion. Maybe it's true, I didn't delve into it. A possible solution is perhaps the centrifugal force of the entire galaxy or perhaps the particles themselves also revolve around the sun like any other substance, i.e. there is no reason why gravitational pushing should not also act on the particles themselves, the fact that they are small should not have an effect, the size does not matter. And maybe something else? What's more, the solar system has always known how to overcome friction and there is no reason why it won't do so even today according to pushing gravity.
    In general, note that the planets and the moons, most of them all, rotate in the same direction, so there is a reason for this.
    I wouldn't attribute too much importance to the friction problem, but it's food for thought and it will keep me busy.
    Please be optimistic and that this will not cause friction between the respondents.
    So floating point and Israel I hope I have at least partially or temporarily answered the friction problem.
    Good and bright day
    Yehuda

  63. To my father Blizovsky
    My response awaits your decision. For the sake of good order, I would suggest that you give a list of rules that will make it easier for you and us. Say what you expect from a response. For example, maybe its length and the like. Not all the rules because I'm sure you have some secret ones. Another thing, perhaps it is desirable that you choose the names of respondents that you will trust until they get "dirty" once.
    My opinion is also shared by other commenters
    Good day my father
    And keep up the good work
    In appreciation!
    Yehuda

  64. Below is my opinion on the latest comments

    Miracles
    I get relativity but it needs an upgrade in distances.

    Liazzi
    In the Andromeda solar system, the gravity is the same as in our solar system, but the attraction between the two solar systems is different from ours. This is what is meant by the term "at great distances". That is, there is no agreement that if the distance between two stars is thousands of light years, the gravitation between them will be according to Newton's formula.
    to Ariel
    If the universe is full of particles that move from place to place, colliding with each other, then they define a gas, and therefore, they have all the properties of a gas such as: volume, particles per cubic meter, and also pressure and pressure differentials and winds that operate for millions of years and determine facts in the field.
    Shmulik
    I said from a Newtonian point of view they move 1.3 times the speed of light, but I immediately added that from a relativistic point of view they move close to the speed of light. Understand that there is a problem with such a declaration, because I am taking conclusions from an existing scientific theory and accepting them in a scientific idea that I am developing and this does not necessarily have to be the case. That's why I separated and said the two options.
    Ariel
    I didn't change my conclusions according to Albanzo, so please correct yourself. The idea of ​​the pressure difference has been with me for years without anything to do with the Albenzo.
    And as for the elegant explanation of the dark mass, not everyone will agree with it. For example, donkeys will not agree to eat only dark bran even if you tell them that it fills the entire universe and is as elegantly nutritious as brown bran.
    To Israel
    The dark matter is supposed to be found from gravitational calculations mainly at the edge of the galaxy or in the gas clouds outside it. Near the Earth it is "thin".
    to floating point
    Are you talking about patience??, the dark matter has been searched for eighty years, without much success, I have no problem if you want us to wait patiently for a few more hours!
    good day everybody
    Sabdarmish Yehuda

  65. Shmulik,

    Regarding both things.

    1. Let's start with the video. The first impression is, of course, that Neil DeGrace-Tyson (despite his enormous contribution to popular science and bringing physics to curious people who don't necessarily have the mathematical knowledge necessary to read articles), is simply turning the whole story into one big joke and putting his need to entertain far, far ahead of the panel . I'm not just trashing him, it has great relevance to what I'm going to say: even though the video is 12 minutes long, the topic you asked me about barely takes up half a minute. There is very little information, so it is difficult to answer. But in principle - noise and ringing. Tyson really enjoyed playing to the crowd and wowing them, but in the end you saw that nobody got too excited, and the reason is simple. There are many equations that can be written, and among many of them there are connections (transformations) that transfer solutions of one to the other. The mere fact that a certain algorithm (there are many) that turns an equation into something visual gives similar results in two cases, does not mean that there is a deep causal connection between the physics behind the equations in question (note also that he claims only that the graphics of the equations are similar, Tyson is the one who turns this claim into " the same equations"). It is possible that the two equations have a certain symmetry that links them, but that does not mean that the universe is a code. Additionally, there is a huge difference depending on which equations it shows. For example, if it presents equations of motion (which tell us how a physical particle should move) or an equation that describes the interlacing entropy in a supersymmetric theory. In the second equation there is a lot of human factor - a lot of things that we have defined (in fact, the whole concept of entropy is artificial. Very useful, but artificial, like the concept of division for example). In this case it is possible that the relationship between the equations is at all a result of our definitions. Finally, we should comment on the side of error correction: I assume that the equations he is talking about are optimization equations of one or another algorithm. Similarity can arise from the fact that it is an optimization, or from the choice of the algorithm. In conclusion, similarity between equations (or between their graphical presentation) does not have any a priori meaning in physics. There may be a million and one reasons for this, and most of them are purely circumstantial.

    Regarding Carroll's language, I haven't read the article, but already from reading the blog it is clear that there are some things here that should be noted. First, there are a lot of semantics here. Note that he explicitly says that the models of small disturbances in inflation are also accurate according to his method, he simply does not call them fluctuations of the quantum vacuum, but decoherence that arises from the multiple universes (and the expansion in them and the temperature change) which in practice causes a measurement of a statistical nature. Mathematically there is no difference here, and this would also be true (in my understanding, there is a place to read the full article of course) even if we conformally connect a huge universe to a tiny universe, which is what Penrose is talking about. I have a feeling that it won't matter to the picture of Lawrence Krauss or to other pictures of the emergent universe, but again - you have to read and I didn't have time for that until now.

    In addition, the differences that Carroll does mistake to exist, derive directly from his assumption of the development of a quantum state in time, which results from his support for a certain interpretation of quantum mechanics (specifically, a certain case of multiple worlds). Which means that if he is right, it is very nice to find descriptive differences between interpretations, but it is not clear how such a model can be built without knowing if the interpretation is correct. I mean, there's some kind of circle here and it's not clear how it's broken (at the moment, I'm not claiming that it can't be broken).

    Sorry I didn't help more, but to give better answers you need to study the subject (which is always true, and I wish it was as clear to everyone as it is clear to the overwhelming majority of people). This week I will try to delve at least into the topic of Carol (which is much more interesting to me) and maybe I can expand. In the meantime if you have specific questions, you can try to ask although I will probably answer that to answer I need to read the articles and go through the math.

  66. Truly Israel, the Higgs Boson has been searched for 40 years, and they still do not guarantee 100% that they have found it. The neutron was searched for 20 years.. Are you sure that "it is impossible to find some"? A little patience 🙂

  67. jubilee

    If according to you the dark matter is inside the solar system and it constitutes such a significant percentage of the total matter in the universe, why is it not possible to find some here in Israel or on the way to the moon for example?

    It's hard to argue with an invisible Dunedin who outweighs the mirror by 70 kilos.

  68. Yehuda, initially you claimed that the problem we have is that at large distances gravity as we describe it (the square of the distance) is incorrect. After Albenzo showed you that changing the exponent would cause more problems, you also added pressure difference and gravitational pushing to the equation (even though this is contrary to your reservation for dark mass) and you still ran into problems (again, many thanks to Albenzo). Remind me how this is more elegant or logical than adding dark mass to our calculations?

  69. After we talked about dark particles that are 1.3 times faster than light (!) that are not the dark matter(!), we learned that the dark matter near a massive body is the one that is responsible for the bending of the light rays and not the massive body and the spaces between the particles are the medium that carries the electromagnetic waves, we will talk about alchemy and its various uses in overcoming the second law of thermodynamics.
    Will be interesting

  70. Yehuda, could you please explain to me what you mean when you say pressure difference? The explanation you gave was not so clear to me.

  71. We have no reason to assume that in other galaxies other physics are arbitrary, but since the galaxies differ from each other in size and mass, we are allowed to assume that each galaxy has fixed physics that are unique to it.

  72. Yazi, for your understanding that gravity is a product of a body/bone:
    The existence of a body is a product of local density of the dark matter, as is the existence of gravity. Let us understand, it is not the existence of a body that creates gravity, but the existence of gravity and the existence of a body are both created at the same time from a third factor - aka the dark matter.

  73. go ahead
    The Earth pulls the Moon with the same force that the Moon pulls the Earth. Both rotate around each other, with the axis of rotation at the common center of gravity. And it's more complicated than that, because the movement is elliptical.

    Everyone agrees that everywhere, at short distances, the law of gravity is Newton's law, that is - the force depends on the masses and the square of the distance.

    The question is whether this law is also valid at large distances. The question is asked - because we see strange phenomena at great distances. The accepted explanation is that there is additional matter that we do not see directly - and this matter has mass. There are other phenomena that this substance explains, and we know many of its properties.

    Another explanation is what Yehuda says - Newton's law of gravity is not valid at large distances.

    I'm not a physicist, but when 99.9% of physicists say something, I bet they're right. And it is especially true that the competing explanation seems illogical to me.

  74. go ahead
    "Because it is more massive, it attracts the moon and not the other way around."

    Also the other way around, but only with one sixth of the force that the earth pulls the moon towards.

    "Does it mean that in Andromeda gravity works differently or doesn't exist at all or something else?"

    I don't think this is what Yehuda claims. But if we treat gravity as a pushing force - which is what the Pushing Gravity theory claims - it can logically shed light on the rotation anomaly of the stars at the edges of the galaxies, which is the trigger for the dark matter theory, even though there are other reasons justifying its existence.

    The main problem that our Yoda agrees on is that it is still impossible to go into a supermarket and buy 2 kilos of dark matter and a basket of gravitons.

  75. Judah, Israel, Yuval
    According to my understanding, gravity is a product of a body/bone. The more massive the body, the more the body produces a force that attracts other bodies (with a weaker mass than it) to it.
    Between the Earth and the Moon there is a gravitational force that consists of the mass of the Earth and the Moon. Since it is more massive then it attracts the moon and not the other way around. Of course, other heavenly bodies also participate in the process, such as the sun for example.
    That is, it does not matter if it is here or in the Andromeda galaxy, between the bodies there is a force of attraction that arises from and because of the bodies themselves.
    In my understanding, even an elementary particle has a force that attracts another particle to it.
    What I can't understand is what you mean when you say "at great distances".
    Is it meant to be an intermediary between one body and a second body? Does it mean that in Andromeda the gravity works differently or doesn't exist at all or something else?

  76. Yehuda
    It doesn't work for me. In general relativity, as I understand it, the force of gravity does not end at any distance. Leave for the moment observations that show that there are other phenomena - the theory of relativity says something very specific.

    You either accept the theory of relativity or you don't.

  77. what is happening to you 🙂
    In circular motion there is a centrifugal force. In our case gravity is the centripetal force.
    Centrifugal force is an imaginary force, have you forgotten?

  78. Israel, for your wondering "We are just trying to understand how it is that the Earth pulls the Moon if there is nothing in between, and Pushing Gravity gives a beautiful and clear description. But there is the main problem of friction, and a host of secondary problems", two things:
    A) The fact that we have not yet found that there is something in the middle does not mean that there really isn't. The existence of dark matter in the intragalactic space is already an observationally proven fact, and there is no reason to assume that it is not also found within the solar system and in all areas of the universe in general.
    b) The phenomenon of attractive and repulsive forces, as well as the phenomenon of friction, are mathematically defined in known physics. Trying to define themselves using them is circular and unnecessary.

  79. Yehuda, you wrote: "There are three forces at work here, frictional and centrifugal gravity, and the movement will be the balanced vector of all these forces. I don't see a problem." If the balanced vector does not point in the direction of the center of the circle, there is definitely a problem, because the circular path will not be preserved. And you have a mischievous force called friction and it is perpendicular.

  80. Israel was ahead of me in the matter of friction, but Yehuda, I have to refine something..

    You wrote "The centrifugal force of the planet, for example, will be equal to the resulting gravitation minus the friction." Have you forgotten what friction is? Centrifugal force is perpendicular to the direction of movement, friction is with the direction of movement! You cannot subtract friction from gravity if they are perpendicular to each other.

    Israel is right - to overcome the friction there must be an additional wind with the direction of movement and as he wrote, this creates an absurd situation.

  81. To Israel
    I don't agree with you. Sailing ships are also able to move against the direction of the wind.
    There are three forces acting here frictional and centrifugal gravitation and the motion will be the balanced vector of all these forces. I don't see a problem.
    We must apparently do an experiment
    Well it's time to sleep
    Good night
    Yehuda

  82. Yehuda

    A ship sailing in a circular motion needs a source of energy to overcome the friction with the water.

    If according to you the "wind" is the source of the energy, it must be exactly in the direction of the ship's movement, which is impossible because every other ship, including those sailing in the opposite direction, will need wind in their direction.

    Another thing is that friction generates heat, even in the example of your sailing ship, and heat is energy that has to come from somewhere.

    In the example of the ship, the wind is the source of energy. This cannot exist in the case of Lasage particles, because, as I mentioned, circular motion in the opposite direction will also consume energy from those particles.

    jubilee

    Where did physics come into the world - a respectable question for simple mortals like us.

    We are just trying to understand how it is that the Earth pulls the Moon if there is nothing in between, and Pushing Gravity gives a nice and clear description. But there is the main problem of friction, and a host of secondary problems.

  83. Israel Shapira
    According to this logic, sailing ships will not be able to sail in the sea because they have friction with the water and the air???. The reason they still don't stop sailing is because the wind is constantly blowing and overcomes the friction, therefore even in gravity pushing, as long as the particles continue to reach the system, they will continue to create gravity that will succeed in overcoming the friction. Hope I don't have to explain it again.
    to Ariel
    Your first question
    There are not easy but possible ways to prove the ideas that lie in pushing gravity or at least about the incorrectness of the square of the distance of gravity at large distances. For example the double system double epsilon lira is supposed to rotate more slowly than according to Newton and a few other options
    And about the second question
    The pressure difference that logically exists in a universe full of moving particles can create motion without having to gravitate mass. A pressure difference can explain the accelerated expansion of the universe without the need for dark mass, but that's for a whole lecture.
    Good night
    Yehuda

  84. Talking about "friction" or "absorption" of "gas particles" is an attempt to define physics using physics. This is a circular definition that will always leave a taste of "more" and will not solve the question of how physics came into the world.

    At the time I had an argument with Yoav Litbek who brought a beautiful pictorial model that claims to explain basic physical phenomena through complex physical phenomena. The argument between us ended in a breakup, which I regret. But the point I insist on is that the laws of physics known today are the result of basic laws of physics that are still unknown to us.
    On one thing I am ready to agree, and that is that even the basic laws of physics that are not yet known must answer to any mathematical quantification formulas.

  85. General relativity is a model that quantifies physical phenomena but does not explain them.
    The "curvature" of light in its passage near a massive body is explained by a density cascade of the dark matter. Great density is expressed in the bodies known to contemporary physics; Low density is expressed in what physics today recognizes as "empty space". The dark matter particles build physical bodies; The spaces between the particles are the medium that carries the electromagnetic waves.

  86. Yehuda

    A body moving through the air, including in a circular motion, will slow down and stop after an extended period of time.

    As Feynman pointed out in the lecture you gave, the Earth had to slow down and stop in the 4 billion years it orbits the Sun. It doesn't happen, he doesn't even slow down.

  87. Miracles
    The theory of relativity does limit the range of gravity but it does so with the dark energy which at large ranges is more dominant than gravity and its opposite direction.
    Try another example.
    Good evening, miracles

  88. Yehuda I have two questions for you:
    1) Is there a way to test the correctness of your theory? (Maybe a nice prediction on the side or an explanation for a phenomenon for which we still have no explanation)
    2) You claimed that adding dark mass to our current model of the motion of galaxies is a tricky way to make our theory fit the experimental results. How does adding additional models (gravity pushing and pressure difference) circumvent the problem you described? (As it seems, you only complicated matters)

  89. Miracles
    Maybe this will surprise you but I will explain it with the theory of general relativity. I don't oppose her. If there is a problem that I can't explain with pressure differences with relativity and pushing gravity and I have to add dark mass then I will gladly throw up my hands and apologize to everyone. Meanwhile there is no need for dark mass. Bring a practical example and then we'll see if we overcome it.
    good evening
    Yehuda

  90. Yehuda
    How do you explain the bending of light rays, or the slowing down of time in a gravitational field, without general relativity?

  91. Israel Shapira
    Who said there is no friction in the Pushing Gravity system?, there is friction but it should not interfere with gravity simply the centrifugal force of the planet for example will be equal to the gravity created minus the friction.
    Miracles
    Why is it difficult for people to understand that the dark mass is the change in the measurements so that they fit the gravitation formula even at large distances of the galaxies??
    Strong
    I believe there is no gravitation at the great cosmological distances and if a contradiction arises from this then someone should back off with their theory.
    good evening
    Sabdarmish Yehuda

  92. Yehuda
    As far as I know, the assumption of the theory of relativity is that gravity does not change with distance.
    The same goes for the quantum theory.
    One sees the force of gravity as a result of the curvature of space, the other sees it as a force carried by theoretical particles, but both do not refer to changes in gravity at large distances (apart from the decrease in relative influence in relation to the distance between two bodies)
    If you claim that the force of gravity is different from what is accepted, or that there is "pushing gravity" (I admit that I did not understand if you see this as an additional force or as another manifestation of the force of gravity), then you are arguing against these two theories and it is appropriate that you provide explanations for this.

  93. Yehuda
    You can object to the dark matter hypothesis, but to claim that someone is trying to change measurements is a pretty serious accusation. Are you claiming that there is an article that someone faked?

  94. To firm
    I didn't move the issue to personal lines.
    I do not compete with relativity and quantum theory. Where did you see that?
    SA I oppose the dark mass and energy and see them as an attempt to change the measurements obtained in the field so that they fit the formula.
    You also claim that I am going against established theory. May I know what theory you are talking about? About mass and dark energy??, sorry, my opinion is not yours.
    And as for the "pit" and other epithets, I think he exaggerated.
    And as for expressing an opinion in an anonymous system, then he is anonymous, I am not!
    Good Day
    Sabdarmish Yehuda

  95. Yehuda
    I have no tools to respond to your speculations. I'm not a physicist and my training on the subject ends with an introductory physics course A and B.
    However. I am able to detect evasion of an answer and moving the topic of the discussion to personal lines.
    If you have a theory that competes with the theory of quantum fields and the theory of relativity, it is appropriate to provide some formulaic basis for the theory, to refer to the problems that exist in the theory and to observations that verify or conflict with it.
    When going against an established theory, it is appropriate to provide explanations of the sequence of hypotheses that led to the construction of the theory and to refer to the reasons that cause the scientific community not to accept this theory.
    Also, ignorant in a scientific context refers to the lack of knowledge in a certain field. Not as a curse as is common in everyday language.
    This is not an insult, not to mention grounds for a lawsuit (even if it was not an expression of opinion in an anonymous comments system)

  96. Yehuda,

    "Going down to my level" does it really mean to deal objectively with the fact that the theory you are trying to sell to everyone simply provides predictions that are found in the laboratory to be incorrect?

    you are ignorant And threatening stupid lawsuits won't help anyone and won't change the fact that you're talking about things you have no idea about. You don't know what you're talking about, so you are - quite literally - ignorant.

    Threatening lawsuits only adds insult to injury. You see, no matter how much you don't like my language (I think the real reason you're so excited is not my language, but the content of my words - you know you have nothing to answer for when you're exposed as talking nonsense that was debunked over a century ago), I've never I tried to shut you up. I never tried to drive you away. I never threatened you. All I did was make sure that every person who reads your "guessings" knows that scientifically they are a big pile of bullshit. You're trying to shut me up, and it shows things about you that are far worse than just ignorance.

  97. albentezo,
    Obviously. Usually it's rude to jump a question to the top of the thread but because of, hmm, "the load" I decided to flood again.

    Thanks in advance

  98. for everyone
    I won't answer Albantezo anymore, I don't want to descend to his level. For his sake, let him be very careful with his words so that I don't sue him.
    Calling me "ignorant" and the like may not pass in silence.
    Yehuda

  99. Yehuda,

    First, I assure you that I am not solid. Eitan is indeed right in what he tells you, but he misses the main point: you are an ignorant person (and there is no insult here: you yourself admit that you have never studied the subject of gravity beyond the XNUMXth grade formula, that is, you have no idea about general relativity, in the theory of dark matter , etc.) who tries to impose his ignorance on reality..

    And of course, what will an ignorant and cowardly person say when they put in front of him rebuttals to the nonsense he says? "I'm not going to address that. I'm sick of". Of course you won't respond. Because you can't respond. Because you insist and argue about the correctness of a theory whose number of refutations does not fit in a response or in ten responses on the site. And you insist on it because of your ignorance. If you bothered to study the things you are talking about, you wouldn't be saying what you are saying. You see, instead of accepting the obvious explanation (that the reason everyone who understands gravity rejects this theory is because once you study the totality of the evidence and facts you see that the theory simply does not describe our reality), you resort to paranoid explanations ("I'm the only one who thought of this! Physicists Professionals and successful people cannot understand what I am saying! There is a conspiracy! There is a conspiracy!”), and this is because you are an ignorant and poor man who lives in denial. For evidence, you think that experience in managing a factory optimization business qualifies you to talk about advanced topics in modern physics. That's it, there's no more delusion than that.

  100. To the new responder Eitan

    In one response you did the following:
    1. You started by being underestimated-
    Answer:- Except for one, everyone responds with respect. Agreeing with the other's opinion or not, it is legitimate.
    2. You have stated that you are ignoring a factor that creates a problem for your hypothesis without explaining why you are ignoring it
    Answer: After I explained it several times, I will explain it again, go, Ethan. In the gravitation of the solar system, for example, the gravitational force is equal to the centrifugal force of the planet. But the force of attraction is not only gravity but is the balance of gravity, solar radiation and... friction. After all, different particles are always moving around in space. It never stopped the planets from rotating and there is no reason for it to stop if the explanation is actually the pushing gravity.
    3. You claimed that you won't even try to explain because one commenter here, in your opinion, is unable to understand the explanation-
    Answer: - True, in my opinion the respondent does not want to understand or is unable to understand.
    4. You acknowledge that your hypothesis lost validity at the beginning of the 20th century but does not address this
    Answer: On the contrary, I think it is time to check such ideas every few years or months.
    I am also testing other ideas that have been accepted for decades, and which may surprise you, for example that the speed of light is not constant, that the moon moves away only at 25 mm per year and not 38 mm, that the weight of bodies changes as a function of time and of course there is no dark mass and energy. And on and on. So what if others say otherwise?
    5. Blaming others for blaming you for their problems
    Answer:- Our wise men have already said: the disqualifier, in his own right, disqualifies. That is, when you accuse someone of something, check yourself first.
    6. And finally, you refer to yourself in the third person-
    Answer: I didn't see where, and if so then why is it so important?
    If someone were to present their arguments in front of you like this, would it be easy for you to take them seriously?
    Or maybe you would ask him not to let ego considerations influence the way they are presented?
    Answer - I have never said anything without thinking when only ego considerations guide me. In science, ego does not determine. In the business streamlining I was involved in, I would approach the most ordinary workers in the production process and ask them what they think the problem is and you will be surprised, these "stupid" workers usually knew what the problem was and offered good ideas for a solution. My ego considerations are very, very, very far from me.
    I hope I answered your questions
    and hope you're not an albanzo (although what does that matter)
    good week
    Sabdarmish Yehuda

  101. Shmulik,

    Sometimes I'm not around for a few days (or a few hours) and suddenly a lot of comments accumulate, so I don't have the energy to catch up on everything I missed (or want to). In the future I will go back and look at what you asked, hopefully today (but no promises).

  102. Yehuda,

    1. Your reading comprehension is just a joke. It is pitiful that a person reaches old age without being able to understand a simple text. Don't worry, I'll explain exactly what you didn't understand (although the catch-22 here is that of course you won't understand what I'm explaining even now):

    2. At no point did I say that you are unaware of friction. I was only referring to the fact that you claimed he was not a problem. So what to do, Yehuda, is a problem. When a theory gives a predication and then you go to the laboratory, make observations, and see that the predication clearly does not materialize, that's a problem. Maybe not in the fantasy world of retirees with XNUMXth grade physics knowledge, but in the world of science - a problem.

    3. It's funny that you write "Even Richard Feynman wrote it!" And thus attributes to me as if I gave special importance to Feynman's words when in reality I wrote in black and white that Feynman is only an example (which I use because you brought it up), but in fact anyone who has bothered to research the subject in the last hundred years knows that it simply does not correspond to observations.

    4. If you insist on not proving your words, but simply repeating like a mantra the absurd nonsense (absurd in the mathematical sense) that friction is not a problem, then it shows that you think there is democracy here. The fact that you say "I am entitled and able to disagree that the friction is a problem" indicates that you think that everyone has an opinion and every opinion has value, which is more or less - democracy. What to do, in science it is not like that. If you say something without being able to provide evidence for it then your opinion has no value. In case it's not clear enough, I'll say it again - Yehuda, your opinion has no value. And it's not because your name isn't Feynman, it's simply because you just talk nonsense and you don't have the ability to substantiate your nonsense as required by the scientific method: with a mathematical model that provides predictions that match reality. What to do, the predictions of the model in question are completely contrary to reality. I invited you in the previous response and I invite you again - please present a mathematical model of the aforementioned Torah that does not create friction (that is, does not predict predications that do not exist).

    5. "I won't try to explain" = I'm too cowardly and stupid, so I'll make up an excuse that supposedly excuses me from justifying my words. And yes, Yehuda, of course I can't understand. Of course... why would a professional physicist (and even if I may testify, excelling in his field) who has been researching gravity for years understand something trivial in classical mechanics? It is clear that you need at least 40 years of experience in managing a plant optimization business to understand a classic argument regarding "why friction doesn't matter".

    6. And again we come back to the fact that you can't read at the level of a XNUMXth grader. You accuse me of not understanding why mechanical force is proportional to mass, when in my response I *explicitly wrote* the explanation for this. Say, are you stupid or what? I specifically wrote the explanation for this, and what to do - it only works in certain cases (which are the cases that were known two hundred years ago, when these theories were written). But this explanation doesn't work for particles that have no mass or charge, even though we see in observations that they are affected by gravity (observations, remember? That thing that in your eyes the Torah should not agree with).

    7. I never underestimated Shankar. As usual, your brain is talking nonsense. I underestimated *you*. I underestimated the fact that there is a fool in the world who thinks that a degree in management and experience in streamlining factories makes him qualified to deal with dark matter. I have good friends who studied at Shankar, and they are excellent designers. I despise you just as much as I would despise someone who studied biology and neuroscience at MIT and thinks that qualifies them to study dark matter. I don't look down on biology or M.I.T., I look down on a person who thinks it's possible to research topic X without studying topic X. Your case is obviously worse, because your "qualification" isn't even academic. I mean, for the benefit of the neuroscientist I mentioned earlier, he should at least say that he understands what research is, that he has experience in studying nature, raising hypotheses and disproving them, conducting experiments, reading articles, etc.

    8. I'm not "saying" you don't think logically. I'm showing exactly what doesn't make sense in the mix of nonsense you spew. Unlike you, I *never* give a claim and refuse to back it up with evidence or explanation. I accuse you of stupidity and irrationality because I reveal in every response that you say nonsense.

  103. Yehuda
    In one response you did the following:
    1. You started by being underestimated
    2. You have stated that you are ignoring a factor that creates a problem for your hypothesis without explaining why you are ignoring it
    3. You claimed that you will not even try to explain because one commenter here, in your opinion, is not able to understand the explanation
    4. You acknowledge that your hypothesis lost validity at the beginning of the 20th century but does not address this
    5. Blaming others for blaming you for their problems
    6. And finally, you refer to yourself in the third person

    If someone were to present their arguments in front of you like this, would it be easy for you to take them seriously?
    Or maybe you would ask him not to let ego considerations influence the way they are presented?

  104. to Albenzo

    Sometimes it's annoying how a responsible person like you reads things in a flash, because it's possible to throw dirt on Yehuda and belittle the commenters just like that without going into depth.
    If you were to delve a little deeper into my response you would see that I am aware of the friction but do not think it is the problem.
    And you will be surprised, I am allowed and able to disagree that friction is a problem, even though someone as great and great as Richard Feynman said it! And there is no point of democracy here.
    I will not try to explain to you why the friction is not important for the gravity of the pushing gravity because you do not want to understand and it is contrary to your conception to understand and maybe you are not able to understand either!
    You also won't be able to understand why the collision of the particles is actually proportional to the mass and not to the surface, because you don't want to understand, and it's a shame because despite everything you have a lot of knowledge and a sound head on your shoulders that could have contributed.
    Do you think that I don't read that the theory was already disproved years ago??, but I learned from Shanker (remember??, that bird you were so scornful of??) and even before that from the kindergarten teacher Esther at the kindergarten "Bambi" that if you haven't dealt with a certain subject for several years then you must That the time has come to take care of it", so you are talking about a hundred years that they did not innovate anything on the subject of pushing gravity??
    But of course you say that I don't think logically, that I don't want to understand, etc., etc., etc. Hey guy, you know the camel can't see its own hump??

    All the best to you and to all the people of Israel
    good week
    Please respond gently.
    Yehuda

  105. For Yuval Khalkin
    I'm not sure there is such a list for my father, even the old me sometimes waits patiently for a response confirmation
    Maybe it's really desirable to have such a list!
    Only after someone "gets dirty", impose censorship on him.
    good week
    Yehuda

  106. 1. I didn't read the Wikipedia entry, but I did see Feynman's reference. What he is talking about there is exactly the mechanical theory of gravity of Lesage and others.

    2. "Feynman cancels it because of friction, which I do not agree with." What is this, democracy? What does "disagree" mean? If Feynman is wrong, prove it! (By the way, this is not Feynman. Already fifty years before him, people realized that this idea simply does not correspond to reality in any way). Please see a model of mechanical gravity as described in the video that *does* not produce frictional forces.

    4. Of course, friction is not the only problem, it is simply something really trivial that can be explained in one line and clearly see that the theory falls because it does not fit reality. There are many more discrepancies between the idea in question and our reality. For example, in the mechanical theory, gravity is proportional to the surface area of ​​the body experiencing gravity and not to the mass. It was customary to solve this problem by saying that the particles in the flow are very small and therefore actually penetrate the body and interact with the building blocks, which have a constant density and therefore receive a proportion to the mass. But this is simply not true in microscopic scales, and in an even simpler way - not true for massless particles. Moreover, today we know that there is no such thing as "mechanical force" - all mechanical forces are the result of averaging over electromagnetic interactions. Thus, if we understand the particle flux as exerting a mechanical force, it will not interact with uncharged elementary particles (for the same reason that a photon cannot be pushed). We of course know absolutely and measurably that uncharged and/or non-massive particles feel gravity. And the list goes on and on…

    But what to do, when a man arrives who avowedly refuses to study a certain subject before he talks about it, and tries to sell on popular websites theories that were empirically disproved more than a century ago, it is hard to expect him to listen to such trivial things as facts or logic, or to take minimal steps to understand what he is talking about Speaking, it is easy to prove his claims (or at least try to prove them. It is impossible to prove, because they are wrong).

  107. I also publish my comments here in the parallel universe of Facebook. I will continue to do so until my name is put on the list of exemptions from the gift
    Thanks in advance, my father

  108. Gravitation is a process that occurs within any space inhabited by dark matter and manifests itself in the observed phenomenon of bodies being attracted to each other. Newton had already come up with a formula linking a property of bodies, known as mass, the relative distance between them and the acceleration with which they approach each other. In theory, this formula is good and correct for any mass and any distance - individual atoms like galaxy clusters, but within structures called "atoms" a deviation from the formula is revealed, and this requires a good understanding of how the attribute mass is created from dark matter.

  109. To all Man Dibai
    I did read Le Sage's theory where it is explained how he sees the formation of gravitation and it is different from my way. Perhaps this is why he concludes that the speed of the particles must be 100,000 times the speed of light.

    The path of many others and mine is explained in Richard Feynman's lecture at Cornell University starting from the eighth minute.
    https://www.youtube.com/watch?v=kd0xTfdt6qw

    • Later, Richard Feynman dismisses it because of the friction which I do not agree with.
    Anyone who wants to have a look.
    And Yuval Hankin, if we don't have a problem with the temperature of the coffee, then you're probably welcome. Goodbye!
    good week
    Yehuda

  110. Thanks Yehuda
    I like my coffee cold 🙂
    During the long wait for my comments to be thawed I discovered that apparently there is a parallel universe on Facebook and I recommend that we all spend our time there as well
    goodbye (Y)

  111. Hello to Yuval Chaikin
    How are you?, you are of course still invited for coffee but take into account that it is getting colder because the whole universe is cooling down and in addition the vanity constant and the (accelerating!) expansion of the universe make the meeting between us more difficult.
    All the best
    Sabdarmish Yehuda

  112. Israel Shapira

    Yoda, let's try this:
    1. If the average distance of the particles from each other is large and they collide with each other - we will get gravitation, but only at large distances.
    This contradicts what we know about gravitation even at small distances.

    Answer: No!!!, only at small distances will there be gravitation!. The collisions of the particles among themselves make the movement of the colliders random and therefore the information contained in them at large distances becomes random.

    2. If the distance is small, we got a simple gas.
    In such a case there will be no gravitation at all, because there will be no pressure difference, as there is no gravitation between 2 bodies in the air.
    Answer: You are wrong again. Between two close bodies (several cm) in the air, gravitation will not occur because the average free path of the gas molecules is very small on the order of micrometers and therefore the particles lose the information about the gravitation that they carry in their directed movement. If the gas will be thinner and the average free path will become at least tens of centimeters in size, then the pull will appear.

    Regarding section 3, I cannot explain it to you any better and unfortunately we will have to leave something for a meeting between us.
    It seems to me that we understand pushing gravity differently from each other

    I finished
    Going to eat and rest. Bye
    Yehuda

  113. Yehuda,
    The standard model does not contain dark matter. Maybe you should read a bit about him.
    Field theory tells you at what energies it will be possible to locate a potential particle that has not yet been discovered. Guess what? Such an undiscovered particle will either barely interact or its range will be much shorter than that of the strong nuclear force. Such a particle is called dark matter and it will not do what you want it to do

    In addition, you are very comfortable with particles whose speed is 1.3 times greater than the speed of light. I'm sorry, it's just not serious

  114. For miracles and miracles

    I got different results than on Wikipedia. Its Newtonian speed is 1.3 times the speed of light and in a relative calculation it is close to the speed of light, and the mass of the particle is close to ten to the power of minus 36 grams.
    But, I will also be gentle, and if you and Nisim are happy with the standard model where one can always add as much (dark) mass as is needed and as much as is missing and get whatever result is required then I am happy for your happiness!
    Anyway, I will look for the article where your conclusions about the speed and mass of the particle are written and then I will respond again. In the meantime, all is well.
    Please respond gently
    Yehuda

  115. Yoda, let's try this:

    1. If the average distance of the particles from each other is large and they collide with each other - we will get gravitation, but only at large distances.

    This contradicts what we know about gravitation even at small distances.

    2. If the distance is small, we got a simple gas.

    In such a case there will be no gravitation at all, because there will be no pressure difference, as there is no gravitation between 2 bodies in the air.

    3. If the particles do not collide and do not interfere and generally do not interfere with each other and the distance between them is small - we will get gravitation according to the Lasage model at any distance.

    But then we return to the basic Feynman question, which I have been asking you for years: what about friction?

    You cannot dismiss this question with a large distance between the particles, because then we return to Scenario 1, that is, gravitation will not be created at small distances, which is contrary to what we know from experimental results.

    So how does it work out?

  116. albentezo,
    If you have patience, then I have two issues that I would love to hear your opinion on.
    One talks about the following post by Shawn Carroll (with your apologies that I keep quoting his words):
    http://www.preposterousuniverse.com/blog/2014/05/05/squelching-boltzmann-brains-and-maybe-eternal-inflation/

    At the time we were talking about a BBC program that presented some ideas about what happened before the Big Bang and one of them was Roger Penrose's who said that when everything was known the universe would "lose" its dimensions and then be re-created due to quantum fluctuation (I hope I'm not wrong here!) . Does the paper by Carroll and his colleagues actually dismiss this idea? Isn't a vacuum, without an observer, unstable? What exactly is a viewer? Is a viewer any *material* that is classical?
    It also links to Lawrence Krauss and Hawking's "Universe from Nothing". Sounds like Carroll dismisses that idea.

    The next topic is about Prof. James Gates said at the Nobel Symposium and also in the attached link about error correction codes that exist in string theory.
    http://youtu.be/cvMlUepVgbA
    Do you know the subject?

    Thanks in advance

  117. Yehuda,
    I'm sorry Yehuda, but your solution proposals are very frivolous, to be gentle.
    TeVeS is an attempt to produce something relative but I sent you to two lectures where they tell you that it still does not solve the problem.
    Nissim tells you that according to the standard model (which is stunningly successful in being compatible with reality) the pushing gravity particles cannot be known particles and according to Wiki they also have to be faster than light (but at some finite speed, which contradicts the relativity that I wanted gravity to be preserved) and you answer very flippantly . It's not that they're hard to spot, but that according to the standard model they don't exist. If they do exist, they are either very, very, very heavy and therefore their range of action is very, very, very short or they hardly interact with normal matter and therefore cannot do what you want them to do.
    By the way, the particles that are very difficult to distinguish are called dark matter

  118. Miracles

    Disagree with your assumptions:
    They are made up of particles with momentum and kinetic energy.
    They are very tiny and hard to notice.
    And the idea of ​​pushing gravity does not rule out relativity.

    good night going to bed
    Bye commenters from all over the world!
    Bye Universe!
    Thanks "Yaden"!
    ()

  119. Although the physical models go well with mathematics, as of now I have not received a response to the question about physics and grocery bills. That is, how it happens that mathematics plays a role in these models.
    The religious among us (it's lucky that it's Shabbat now and they won't desecrate it in order to beat me and beat me) get along well with a dual world where mathematics and heavenly providence are in one submission. I, on the other hand, am always looking for the simple solutions. I do not like, for example, that our universe is populated by so many elementary particles that do not have a common ancestor. Although modern physicists have tried to appease me with the quark model, six is ​​also too big a number for my opinion, what with the fact that each of the quarks has an anti-quark and we have already reached a dozen separate building blocks.
    The model that I have been working on for 40 years (soon to be my birthday) talks about only one elementary particle which already in its early days took the form of what is known today as "dark matter".
    Dark matter is what makes up the particles known to physics. He is also the one who creates the phenomenon of gravitation on the one hand and electromagnetism on the other.
    It would be a mistake to attribute familiar physical properties to dark matter, since physics derives from it, and such an attribution would constitute a circular definition. Therefore, the talk about a "dark photon", for example, is simply nonsense and I strongly embrace the siffa of the article "Obel 3827 will become another example of what dark matter is not".

  120. Israel Shapira
    There will be no gravity in the air because the average free path of the particles (molecules) in it is very small and therefore no attraction will be created. But under conditions of high emptiness it will be possible to conduct an experiment that will show gravitation because then the average free path increases. Do you understand Israel? So who wants to invest in the experiment?

  121. Yehuda
    As I understand it, they can't be made of normal material, because then they would appear in the standard model.

    I also think that not looking for them does not mean that they would not have been found.

    And one last thing - the idea rules out both theories of relativity. It doesn't seem to me that tools that have long since moved from the field of science to the field of engineering should be dismissed so quickly.

  122. Yoda

    If there was attraction in a gaseous mechanism with collisions between the particles, we would get attraction between any two stretched sheets facing each other, wouldn't we?

    Note that without collisions, you do get traction as shown by the value on Lasage.

  123. Israel Shapira

    Why did you determine that gravitational forces are not generated in a gaseous mechanism???, I have been striving for years to do an experiment that would show gravitational forces in gas. Looking for volunteers to conduct such an experiment.
    In addition, La Sage did not comment on the possibility of the particles colliding between them because then he would have corrected the square of the distance for large distances just like I corrected. As far as I know, only I came up with this idea.
    And by the way, the idea of ​​a messenger particle that causes gravitation was already put forward by Newton. He claimed there must be one but he doesn't know how it works.

    Yehuda

  124. Miracles
    These particles are tiny and have a smaller mass than the neutrinos. They have momentum and kinetic energy and are most likely made of ordinary matter. They are apparently like little netrins. I call them my La Sage pieces. They are not discovered because they are not looked for. Not many believe in pushing gravity. But if I am right about the disappearance of gravity at large distances this could be proof of the existence of a gravitational pushing explanation for gravity.

    to Shmulik
    I'm not a follower of the MOND theory, but the renewal takes place from a transition between regions with different gas density that exists in the vicinity of the galaxy and not necessarily because of the proximity of a large mass. I know that Keren Or does this like Betta Morgana or Mirage, but I don't know how it is done exactly.
    In addition, I was not talking about particles faster than the speed of light and I was not talking about the theory of relativity. All I think about the theory of relativity is that it needs to be revised, for example regarding the gravitation at distances.
    Honestly, I didn't delve into it too much. For my part, the respect of the theory of relativity will be preserved. A different explanation of gravitation should not damage the basis of the conclusions of relativity.
    I can go on and on, but it's already after midnight, so please respond gently. The neighbors are sleeping.
    Yehuda

  125. Yoda

    Yuval Chaikin lived in Scotland at the time, now in New Jersey. He is the one who connected me to you.

    Small correction: in Balsage proper there are no collisions between the particles, otherwise you would get a gaseous mechanism, which as we know does not create gravitation.

    I understand your claim that according to your model the collisions, root mean square, are at large distances and therefore we do get a gaseous mechanism, but as I already mentioned to you, in that case you would not get gravitation at short distances, which contradicts experimental results.

  126. Yehuda,
    So pressure difference is a get out of jail free card? Is it possible to simply integrate it into the conversation without being an expert in what is measured in the bullet cluster?

    If Carol's YouTube didn't help, maybe Wiki will?
    https://en.m.wikipedia.org/wiki/Bullet_Cluster

    In theories without dark matter, such as Modified Newtonian Dynamics (MOND), the lensing would be expected to follow the baryonic matter; ie the X-ray gas. However, the lensing is strongest in two separated regions near (possibly coincident with) the visible galaxies. This provides support for the idea that most of the mass in the cluster pair is in the form of two regions of dark matter, which bypassed the gas regions during the collision. This accords with predictions of dark matter as only weakly interacting, other than via the gravitational force.

    I still don't understand how the pressure award can explain this result

    I also don't understand why the need for particles moving at speeds much faster than light is not a problem. How can relativistic effects be explained through this theory? Why does time move slower when flying at high speed? Why does time move slower under gravity? How is prime dragging explained?

  127. for miracles
    You force me to elaborate. Well, the pushing gravity creates gravity by the square of the distance in a really nice way. Richard Feynman explains it nicely in his lecture at Cornell University. Details on Google. But there are two problems with this, the first is the friction that caused Richard Feynman to cancel the explanation. And I claim that it is nothing because it requires a small correction in Newton's formula where: the force of gravity minus the force of friction will give the centrifugal force. Just like the sailing ships in the sea where the force of the wind minus the friction will give the power of movement of the sailing ship.
    The second problem is that as the distance between the bodies increases, there is a greater possibility that the particles collide with each other, and their direction becomes random and therefore loses the gravitational information contained in the direction of its movement. Therefore there is no gravitation (almost) at the large cosmological distances. For further explanation, maybe I will write an article about Pushing Gravity for my father and maybe it will be useful for his publication.

    Please respond gently. It's just science.
    Shabbat Shalom
    Sabdarmish Yehuda

  128. Israel Shapira, are you in the US???... I thought much closer.... I didn't finish crying for an hour. Apparently, the chance that we will meet is getting further and further away, especially since the entire universe is also expanding rapidly with the dark energy!!!!!!!!!!!!!

  129. Ariel does not claim that there is a fundamental logical flaw in Lesage's gravity, but a fundamental logical flaw in Yehuda's process of drawing conclusions. He gave a link to a video where he (supposedly, of course) logically comes to the conclusion that the missing matter in spiral galaxies should be explained by pressure differences. This video is full of nonsense, assumptions, and embarrassing logical errors. Ariel specifically pointed out the fact that on the one hand Yehuda's conclusion is that gravity must be replaced - that what we know about it is only true for short distances and therefore it is actually possible that it is completely different from what we think - but on the other hand he rules out the possibility that there is an intervention of nuclear forces because they only work short term Let alone that it is complete nonsense that we measure gravity only at short distances (there are gravitational phenomena measured even in the largest rocks of the universe), it is clear that there is a twisted logic here. On the one hand - the strong force (measured only at short distances across the Earth) must obey these laws even at long distances and therefore cannot explain the phenomenon, on the other hand gravitation was only measured at short distances so it is possible that it behaves according to other laws and explains the phenomenon.

    Yehuda's logic is completely distorted, a court will not help. He also speaks without knowing the end of the end of the facts, and that will not change either (he also admits this fully, but for some reason he claims that it is not necessary to know the facts to investigate the issue). Lesage's gravity has no logical problems, there is only the small matter of its inconsistency with observations. A small matter, indeed, but still for some reason physicists make a big deal out of it...

  130. Yehuda
    So these particles are absorbed - by what? What do they become? If they are absorbed in empty space, doesn't it make sense that they would be absorbed even more when passing through matter?

  131. Israel I am very well aware of how the strong force works I am only presenting my argument to Judah to show him the flaws in the logic he is using.

  132. Yoda

    United States..

    And they withdrew from the United Kingdom already in 1776.

    And my father's kingdom is science, I'm talking about Facebook. I sent you a request two months ago..

    And the strong force isn't worth much outside the confines of the atom. In fact, if I remember correctly, at some point it reverses and becomes a repulsive force instead of an attractive one..

    Reminds you of something in the context of the article?

  133. Yehuda I was just arguing according to your logic (if we didn't measure in galactic distances then it is not necessarily true) that there is a good chance that it is actually another force (3/4) that affects the movement of the galaxies and not necessarily gravity. I want to know what your arguments are as to why it must be gravity and not some other force.

  134. to Shmulik
    A pressure difference can move galaxies in a cluster without the need for dark mass or gravity at all.
    The beautiful pictures that seem to show the dark mass in the Slingshot cluster, actually show only the areas of inconsistency with the gravitation formula. Pressure differences can explain everything dark mass can. Beyond that, I'm not an expert on the sling cluster.
    Good Day
    Sabdarmish Yehuda

  135. to Ariel

    If you prove to me that the strong force and/or the flying spaghetti monster and/or the angel Gabriel also work over great distances, I'd be happy to use them. Just saying it is unacceptable to me. In principle, since Gabriel and Spaghetti are not scientific ideas, I dismiss them outright. (It is impossible to show a way in which they can be exposed to refutation.)

    To the dear and beloved Israel Shapira

    God forbid I commit such a scoundrel act and not approve the dearest of my friends in the United Kingdom. If anyone has the authority not to approve you, it is my dear father. Apparently you wrote down some "dark" word that didn't pass his censor software. But here is an appearance. Just remember that every morning I reheat the coffee I made for you and with tears in my eyes, I, the infidel, pray that you might finally appear.
    And according to you, I'm a fan of pushing gravity, and I think it's a lovely explanation for gravity, and let's not mention friction because it actually shows the size of my La Sage particles. who know how to overcome friction and create gravitational force. Just like sailing ships that float in the sea despite the friction they have with the water and the atmosphere. This is absolutely not a contradiction!!!
    An exhaustive discussion of my opinion on Pushing Gravity is beyond the scope of this article.

    for miracles

    You are right that there is an "absorption" mechanism, but it will expand the debate in the matter of pushing gravity, which is also in constant denial because of the words of competitors Newton, Richard Feynman and others.
    If the person responsible for the site agrees, I will write an article on the subject. But it was made clear to me that he is not interested in articles that do not meet the scientific consensus. And of course he is the boss.

    So please respond gently
    It's just science
    Sabdarmish Yehuda

  136. to Ariel

    If you prove to me that the strong force and/or the flying spaghetti monster and/or the angel Gabriel also work over great distances, I'd be happy to use them. Just saying it is unacceptable to me. In principle, since Gabriel and Spaghetti are not scientific ideas, I dismiss them outright. (It is impossible to show a way in which they can be exposed to refutation.)

    To the dear and beloved Israel Shapira

    God forbid I commit such a scoundrel act and not approve the dearest of my friends in the United Kingdom. If anyone has the authority not to approve you, it is my dear father. Apparently you wrote down some "dark" word that didn't pass his censor software. But here is an appearance. Just remember that every morning I reheat the coffee I made for you and with tears in my eyes, I, the infidel, pray that you might finally appear.
    And according to you, I'm a fan of pushing gravity, and I think it's a lovely explanation for gravity, and let's not mention friction because it actually shows the size of my La Sage particles. who know how to overcome friction and create gravitational force. Just like sailing ships that float in the sea despite the friction they have with the water and the atmosphere. This is absolutely not a contradiction!!!
    An exhaustive discussion of my opinion on Pushing Gravity is beyond the scope of this article.

    for miracles

    You are right that there is an "absorption" mechanism, but it will expand the debate in the matter of pushing gravity, which is also in constant denial because of the words of competitors Newton, Richard Feynman and others.
    If Avi Blizovsky wishes, I will write an article on the subject. But my father made it clear to me that he is not interested in articles that do not meet the scientific consensus. And of course he is the boss.

    So please respond gently
    It's just science
    Sabdarmish Yehuda

  137. Yes, I don't think the commenters are familiar with the gravity pushing mechanism..

    It might be worth noting that the idea was tested by all the greats, including Newton, Maxwell, Lord Kelvin, Feynman and many others.

    And why don't you approve me on Facebook? Have you already forgotten who served you a small turkey in reserve?

    I also heard that Sage and Zwicky started a WhatsApp group, so don't miss out.

  138. Yehuda
    I mean you assume that space absorbs the bubbles that transmit gravity, right? It kind of contradicts what we assume that space doesn't absorb anything, doesn't it?
    If you think so, you should describe the mechanism of absorption.
    You also have to contradict the other evidence for dark matter.

    Successfully….

  139. Yehuda, then remind me again why you don't think it's not the strong force that suddenly decides to act after a distance of billions of light years?

  140. for miracles
    For every phenomenon that spreads in space as a sphere, such as gravity, sound waves and light waves, etc., there are two factors that affect the spread. A. What everyone knows is the spherical expansion that you also rightly speak of, but in addition there is the disturbance of the medium, for example, light from a lighthouse spreads according to the square of the distance but also according to the turbidity of the atmosphere. Cloudiness that swallows 50 percent every kilometer (due to fog, for example) will leave the light barely per mille than expected based on the square of the distance.
    I claim that as above for any phenomenon that spreads in a quadratic manner including gravitation the question is for what distance will 50% of gravitation be given up. The situation at the moment with Newton is that this distance is infinite, which means that all the gravitation that sets out reaches its destination. I claim from calculations I made that it is about a few light years that will leave only 50% of the gravitation

    to Ariel
    Your second response
    Nicely you said that it was possible to deal with gravity instead, with the circular formula which is actually Newton's second law F=M*a
    that in the circuit is R/F=M*V2
    But this is exactly Prof. Milgrom's idea in his MOND theory, he carried it out without much success. There are phenomena that this theory has difficulty explaining.
    I mean, what I'm doing to gravitation is similar to what Professor Milgrom did to the second law. The conclusions are different.

    to Ariel
    Your first comment
    I imagine that a solar system in a distant galaxy that is similar in size to our solar system would behave in the same way as ours, that is, according to Newton's laws of gravitation for small distances.
    In addition, I believe that a distant galaxy will also behave with regard to large distances of thousands and tens of thousands of light years like our galaxy, but both will not do so according to Newton's laws of gravitation, but according to other or revised laws.
    The cosmological principle cannot increase the proof range of formulas only experiment and measurements can do that!

    Please respond gently
    Sabdarmish Yehuda

  141. Yehuda
    Behind the gravity equation there is a very simple logic - the area of ​​influence of a homogeneous radiation source depends on the area of ​​a sphere, which depends on the square of the radius. If we look at light, the intensity of radiation depends on the number of photons per unit area per unit time.
    Do you think this is also wrong?

  142. In addition, you still haven't answered how you determined that the gravitation equation needs correction and not the circular motion equation.

  143. Yehuda, if you throw the cosmological principle in the trash (which is what you did in your last response) you still have to answer the following questions:
    1) What are the laws of motion that operate in distant galaxies? (After all, Newton's laws of motion are only valid at 10 astronomical units)
    2) If there is no isotropy, why is the temperature of the background radiation and the density of galaxies the same?
    3) Is a distant galaxy that I am observing at this moment behaving according to forces in my region or according to the forces in its region? For example, if gravity is 10 times stronger, does it exert a force on me that is 10 times stronger than what I exert on it? Or do we exert equal forces on each other and if so what is the strength of that force?

  144. to Ariel
    You said: "We cannot conclude that something the size of the solar system will be valid for the entire universe." End quote.
    It's true, but I didn't say it, the English philosopher David Hume said it as well as others: "From what you measure, you only know about what you measure." For example, you checked 1000 books and saw that they are made of paper and they tell the story of Pinocchio, so the conclusion cannot be that all the books in the universe are made of paper and deal with Pinocchio, but only that there are at least 1000 books made of paper in the universe and deal with Pinocchio. And more than that, if something is true in the solar system for ten astronomical units of length, it absolutely does not have to be true for a million or a billion lengths, that is.... If the Earth revolves around the sun due to gravity, this does not mean that the spiral galaxies that are a billion times larger also rotate due to gravity . And please don't quote me the cosmological principle because what the principle says is that in another solar system similar to ours, gravity works. What, to do but that's what there is!, sorry.
    Please respond gently, it's just science (:))
    Sabdarmish Yehuda

  145. Yehuda Why did you determine gravitation? Why not our equations that describe circular motion? Why not let the electromagnetic force begin to affect neutral particles as well? According to your logic, all the forces we have discovered and studied in the solar system can be the factors, as you said, we cannot conclude that something the size of the solar system will be valid for the entire universe.

  146. Yehuda Why did you determine gravitation? Why not our equations that describe circular motion? Why not let the electromagnetic force begin to affect neutral particles as well? According to your logic, all the forces we have discovered and studied in the solar system can be the factors, as you said, we cannot conclude that something the size of the solar system will be valid for the entire universe.

  147. Yehuda, who is the "talented" genius who filmed your lecture on YouTube? The quality is simply horrible, I would be ashamed to put something like this on the Internet.

  148. to the employee
    First of all I really rule out gravitation almost completely at the large cosmological distances. It has never been demonstrated at large cosmological distances. Newton discovers gravitation and claimed that it is defined on the whole universe and since then it has been so. But Newton's universe was small, only the solar system to the planet Saturn, about ten astronomical units. So if it were allowed to say that the formula works on the whole universe. We must not assume this for millions and billions of light years.
    And regarding the pressure difference, let's kill another sacred cow and ask ourselves whether our universe is empty or whether it contains countless particles that move within it such as neutrinos, other cosmic rays, photons, etc. All of these define a gas, meaning that our universe is a huge gaseous body containing stars. And gas has pressures, pressure differences, winds, etc. I argued that after eliminating all other possibilities, only the pressure difference solution remained in conflict with the dark mass idea. And of course we must not forget the void, the dark energy and the accelerated expansion of the universe, everything needs to be explained.
    That's it in short and if you want a more comprehensive explanation, try to find me and call me and I'll be happy to meet and explain my ideas to you and others, you just have to come with an open mind and without prejudices. For example, if you said: "And the truth is, I can't see a solution that doesn't include gravitational effects," end of quote, that's already a decision you'll have to be willing to give up.
    Please respond gently
    Sabdarmish Yehuda

  149. Yehuda, I saw the video and I didn't really understand what your theory means "comparing the pressures"? It doesn't mean much.. maybe you can explain beyond that.. except that I really don't understand.. are you actually refuting the gravitational theory? is that what you say Because it seems to me that you should refine a little what you said there in the presentation, I personally do not know much about the dark mass problem except for the fact that there is a contradiction in the equations that arise from the equation of motion (but what I know is a little longer than the problem as you presented it so simplistically...) and the truth is not I can see a solution that does not include gravitational effects, in general what you say there is a bit unclear, if possible maybe you should call a place that presents what you are talking about in a more academic and established form.. Thanks

  150. I did not understand why there is an apparent contradiction. If anyone knows a scientific article that explains the contradiction, I'd love to see it.

    Those who want to see Carol's explanation about Tebas and the bullet cluster and why playing with gravity without dark matter cannot explain the information that arrived are invited to watch starting at the fiftieth minute on the following YouTube
    (I have already brought it before but Yehuda insists on not seeing or understanding...)
    http://youtu.be/iu7LDGhSi1A

  151. Mr./Ms. does not want to go back to the 16th century
    What will you do in the 16th century?, Newton wasn't born yet, and Galileo hadn't built a telescope yet. Maybe you will get to see Giordano Bruno get on the stake but not sure. And it's also not a heartwarming play worth going back to the 16th century for.
    What's more, if I had been there then, there is no doubt that I would have been very quiet and tried not to annoy the Inquisition so that I wouldn't have my ass pierced with high energies that someone would have organized for me...
    I must point out that the continuation of your comment also interests me and the matter of the neutrino and its speed, often bothers me. Super Nova 1987A exploded in the Large Magellanic Cloud 170,000 years ago and see it's a miracle that the neutrons of the explosion and the light of the explosion reach us almost together. This is apparently a contradiction of relativity!
    So we had something to talk about if we met and Albenzo wouldn't interfere. Maybe we would have invited him as well?….and the kindergartener Esther Magen Bambi??.
    It could have been nice.
    Good night
    Yehuda

  152. At the moment it was published on the website that the winner of the Israel Prize Professor Yaakov Bekstein passed away. I heard him lecture on dark mass and the MOND theory. And he has also been interviewed many times on the subject. He was one of the greatest researchers in Israel.
    sad.

  153. Yehuda,
    But you hear from physicists, including those you haven't argued with yet, that there is overwhelming evidence that no matter how you play with gravity, there are findings that cannot be explained by such a game. Carol explains this against the background of the findings from the Bullet Cluster, explicitly and in easy English, on the YouTube I brought, and you are on your own. I really can't understand why you insist. Based on what knowledge do you contradict the claim that Carroll made? Based on what findings?

  154. Ariel
    You did clarify your point well and let's not forget that in the end Albanzo is the referee and he is the decider and he has connections in the referees union. That's what there is.

    Shmulik
    In the spiral galaxies, they compared gravity to the centrifugal force and discovered that the centrifugal force is ten times the force of gravity. Since they are supposed to be equal between them, add the dark mass. Professor Milgrom's greatness is that he took the existing data measured in the field, and did not change it, but he did change the formula of the centrifugal force - Newton's second law - the law of acceleration F=m*a so that the centrifugal force would be ten times smaller and then there would be equality. Once it fails. The remaining solution is to act in the direction of changing the gravitation formula.
    Good evening and how fun it is to have an air conditioner in the room!
    Yehuda

  155. There is an interesting recurring motif in all science deniers of any kind.
    This argument of - you only tested it for short distances or times therefore it could be wrong...
    The laws of physics are supposed to be universal. The same fields/forces with a single equation that describes them.
    Just because I haven't visited Honolulu, doesn't mean I can't assume the same laws of physics apply there.
    It seems to me that this is what the entire branch of cosmology is based on.
    Despite all that...I have to admit that Sabramish is light entertainment to watch. And I also have all kinds of heretical reflections sometimes about the existence of the Higgs boson. And neutrinos are faster than the speed of light.
    Please do not respond gently.

  156. Yehuda, you may be right and you may not be, but in the meantime don't forget that you are up in the stands and the referee Albenzo is on the field, you may see a refereeing error or a "foul" and Albenzo doesn't whistle, but don't forget that Albenzo is the one on the field and not you, you don't know if the grass Wet, not mowed, etc... and Albenzo actually knows. Of course, you can get this data and then decide if there was a "foul" or not, but until you receive it, you cannot challenge the referee's decisions. In addition, you are also a fan of the same team, therefore your judgment is biased compared to the judge who is more neutral. Hope I made my point clear.

  157. Yehuda,
    MOND is not the reason I recommended you to talk to him (again, in your case) but that he will try to explain to you why no one changes data (meaning no one changes data that comes from our telescopes, etc.). No one knows if MOND is wrong but even if it is good, it is not solved by dark matter.

    In addition, it's a shame you stopped listening because in the second lecture, at about the 40 minute mark, he explains why the Bullet Cluster challenges to the point of absurdity any theory that tries to explain the results only by playing with gravity. It's a shame you came to the trough and refused to drink from it. Renew your missing dark energies

  158. to Shmulik
    Years ago I had the chance to talk with Prof. Milgrom about the MOND theory. An interesting meeting. Unfortunately, it does not pass the proof test. For example, it is unable to explain the behavior of galaxy M94, which has almost no dark matter.

  159. to Shmulik
    I accessed the link you sent me to, and realized that the lecturer was trying to convince that the MOND theory is not a solution to what we call the dark mass problem and since I agree with Mr. Lecturer, I did not find it appropriate to continue listening to his lecture. In these troubled days that I am already far beyond the "Bambi" kindergarten, I should make the best use of the few high energies I have left.
    For your information, besides the MOND theory and the idea of ​​dark matter, there are at least ten other options for solving the aforementioned cosmological problem. Anyway, thanks.
    Please respond gently
    Yehuda

  160. Waiting maybe because I wrote Milgrom's email (from his website at the Weizmann Institute) so that Yehuda can ask him directly about the dark matter

  161. Yehuda,
    Did you see the lectures I referred you to?
    The situation you describe: "You apparently don't understand that the dark matter is the data change made to help the normal matter complete the 90 percent that it lacks to complete the gravity it lacks" is simply a slander of almost everyone involved in this field and I really don't understand why you come to these places.

    I have a challenge for you: Hanan got us Milgrom's site. here:
    http://www.weizmann.ac.il/particle/milgrom/
    This is his email: Moti.Milgrom@weizmann.ac.il

    How about sending him an email in which you lay out your claims against dark matter (and if you're brave write the sentence you opened your last post with)? His answer will be very interesting

  162. to Albenzo
    In section 3 of your response you requested:
    "Give one example of a physicist ever changing measured data to support dark matter." End quote.
    You apparently don't understand that dark matter is the data modification done to help normal matter make up the 90 percent it is missing to make up for its missing gravitation. A wicked change! Those who don't understand this, too bad! I'm sorry, my friend Albanzo, that you look down on the beloved kindergarten "Bambi" and the Shenkar College. Not pretty!

    to Ariel
    God forbid I offend the extensive knowledge that Albenzo has of high, low energies, and/or, at half-mast, what I told Albenzo that this is about: the most basic things in science such as when a scientific idea must be replaced, and whether a formula (for example, gravitation) has proven itself at a distance of less From a thousand light years (the inner solar system) must also be true for distances of millions of light years? This was taught to me by kindergarten teacher David Yom at the "Bambi" kindergarten. These are very basic things and the great knowledge that Albenzo flaunts, his tremendous knowledge of physics, does not belong to the aforementioned basic things and does not add to the understanding of things, and probably even detracts. So of course anyone can talk about dark matter or the beloved "Bambi" kindergarten, but the mental stability of the children of the beloved "Bambi" kindergarten is not related to the state of the high energies in the cosmological universe. So please be precise and don't say that I said he shouldn't!
    In the first class at Shenkar College (or "Gan Bambi" I don't remember that well anymore) the administration teacher (or the kindergarten teacher?) said that you can be very smart and very outstanding, but keep in mind that a person from the settlement or the other grade will always come and find your mistakes. It's really similar to a Beitar Jerusalem football fan looking from the top of the stands at a football game with the best players and discovering mistakes by the referee Albanzo, in the Beitar Jerusalem football game. "La Familia" prove it in every game.

    But, what a beautiful and pleasant day!
    So I'm in a good mood so please respond gently and don't put me down.
    Yehuda.
    Proud graduate of "Bambi" Shenkar and Kindergarten College.
    A fan of Maccabi.

  163. Yehuda,
    Two responses. My original one
    More about MOND and TeVes
    Here is a talk by Shane Carroll where he talks about ideas on how to move beyond general relativity. At about minute 31:20 he starts talking about MOND and Teves and at about minute 40 he gets to the point of why these two ideas fail to get rid of dark matter.

    http://youtu.be/SwyTaSt0XxE

  164. "I graduated from "Bambi" Kindergarten (with honors), I saw the movie Terminator 2, and I have more than 30 years of experience in showering by myself without the help of father or mother. In my humble opinion, this knowledge is sufficient and allowed for the analysis of the structure of the neurological networks in the brain. Actually, I don't understand why I'm not allowed to perform surgical procedures on nervous system cancer patients…”

    Albanzo, you killed me…

    Yehuda, Albanzo's speech may seem very offensive to you, but the message that emerges from it is quite accurate: your education, as you describe it very openly, is completely - but really, completely - insufficient to deal with questions that are three orders of magnitude simpler than the question of dark matter.

    A person needs to know what are the tools he has in his arsenal. You wouldn't go into battle against a battalion of paratroopers armed from head to toe armed with a stick. You wouldn't try to solve the weather prediction problem when you are equipped with XNUMXth grade arithmetic. The subject of dark matter requires depths and horizons that take many years to build, and there are no shortcuts here.

    And it would have been appropriate for you to refer to specific comments: matter and not mass. Matter may have many properties, one of which is mass. A model or theory rather than a formula. In the physics of the 21st century, formulas are not "invented" but the appropriate formulas are derived from models or theories that are based on very solid and very inclusive fundamental principles. "Formula" is just another corner of the model. And after all that, if the calculations and predictions obtained from the model do not coincide with the observations, they are immediately replaced or abandoned (or perhaps parameters need to be recalibrated or, unfortunately, structural deformation) until a match accepted by the entire research community is obtained. No one tries to "smooth" things, on the contrary: the standards of research in physics are very high.

    But, as mentioned, in order to prove this you have to soar much, much higher than the level of high school studies or industrial engineering and no shortcuts will help. Without the takeoff the horizon remains narrow without the viewer knowing that it is narrow.

  165. Yehuda, on the one hand you claim that a physicist who deals with high energies cannot talk about gravitation at all, but a few paragraphs later you state that you can discuss the subject because you are a graduate of Schenker College in industrial management (which as far as I know has nothing to do with gravity). Where is your consistency?!

  166. And one thing I forgot to address - the purpose of the article. The purpose of the article is to tell you about a new phenomenon that is beginning to be seen experimentally, of forces acting between dark matter particles. For that matter, forces in the dark sector, or dark forces. This is the goal, and to achieve this goal there is also a paragraph or two of background. In the background section, they explain about dark matter and also about the difficulties in the theory. If you read what is written instead of covering your ears and screaming, you will see that all the difficulties being talked about are in understanding the microscopic properties of dark matter. That is, although we see dark matter in high concentrations in space, we do not have a single particle in hand that we can perform an experiment on and find out its mass, its charges, the interactions it performs, etc. For that matter, neutrinos also have a mass that we don't know exactly what it is. So in your opinion he doesn't exist, right?

  167. Yehuda,

    Below is a relatively concise reference. Before that, I'd like to congratulate you for doing an *amazing* job (!!!, this cannot be emphasized enough) of ignoring every single thing I said. I can only explain this by an insanely acute case of reality suppression.

    1. I have already explained to you that in science there is no meaning to the impression that comes from the formulations and choice of words of one or another person, especially when this person is a journalist and not a scientist in the field. Continue to depend on the "feeling of despair" that a reporter has (do you understand the difference between a reporter and a scientist? Not sure), because you have nothing else to hold on to.

    2. Does the knowledge of high energy physics not belong to the subject? First of all, you don't know what knowledge I have. My knowledge is not limited to high energies only. Thank God, I also know, work and write about general relativity, which is known to be a limit at low energies in general for quantum gravity. But it doesn't matter, because - how can you decide what is relevant and what is not? How can a person who has no idea what the theory of dark matter is, has no idea what gravity is (beyond quoting a formula you learn in ninth grade), decide what is relevant and what is not? First you will learn the field, then if you want I invite you to test me and my knowledge, and if they are relevant or not. I'll give you a hint: I'm a physicist who deals with gravitation. Gravitation, gravity, gravity.

    3. Give one example of a physicist ever changing measured data to support dark matter. I really don't know if you are a liar or if you just don't understand the difference between a measurement and an idea that comes to you in a dream. I explained in a previous comment - no one changes the data, they change the theory. The dark matter is a consequence of a theory change (in your childish language - changing the formula. Simply instead of changing the exponent of the distance in the formula as you want to do without even understanding what it means, we changed the sources of the gravitational field in the formula).

    4. Do you need a "scientist with balls" to "throw away the gravitational idea"? No. Need someone really stupid. how stupid I will give you an example:

    5. "I am a graduate of Shenkar College in industrial management with honors and I ran a business that dealt with the optimization of factories for over forty years quite successfully. In my humble opinion, this knowledge is sufficient and allowed for the analysis of the dark matter problem."
    This. There is simply nothing to add. A man who is stupid enough to say such a thing, and also stupid enough not to understand how stupid it is, is really not worthy of discussion. I definitely agree that you can cut here.

    Oh, wait - one last thing. I am a graduate of "Bambi" Kindergarten (with honors), I have seen the movie Terminator 2, and I have more than 30 years of experience in showering by myself without the help of father or mother. In my humble opinion, this knowledge is sufficient and allowed for the analysis of the structure of the neurological networks in the brain. Actually, I don't understand why I'm not allowed to perform surgical procedures on nervous system cancer patients...

  168. Yehuda,
    I also tried to show you, in a non-interfering way, through a very nice lecture by Sean Carroll whose topic is "Beyond General Relativity", why dark matter exists. The lecture in English is easy and well explained there, in the minutes I mentioned why Milgrom was unable to get rid of the dark matter. Did you bother to watch the lecture?

    You asked what the purpose of the article is and here you fail miserably in understanding the text and you increased it by cherry picking by presenting the first paragraph as the front of the article. Woe. The first paragraph is dramatic because its purpose is to try to get the reader into the matter and it ends with a question mark, that is, it asks you to continue reading.

    Did you get to the third paragraph? (or the second, depending on how you count)? It opens with the following words:
    "This grim scenario took a hopeful turn in the early spring of 2015. Researchers uncovered one of the most intriguing clues in years: a sign of the existence of a new force that may allow dark matter to "talk" to itself. This insight will help explain what kind of particles, perhaps, dark matter is made of.'

    How about this paragraph:
    "This news excited the physicists and inspired them to look for answers. "If this discovery holds up, then it is much more than sensational news," says physicist Neil Weiner of New York University, who did not take part in the research. A scenario involving dark photons is a modification of the most basic and accepted concept of dark matter as consisting of one and only one type of particle, which is commonly called a "weakly interacting massive particle", or WIMP ("weak", in English). But the idea that dark matter is accompanied by dark photons and exotic interactions may help solve some problems raised by describing dark matter using one type of WIMP, such as why the centers of galaxies are less dense than expected.'

    In the rest of the article the researchers did what you expect from any proper researcher and that is to give the appropriate disclaimers, since it is not about religion but science and how nice of the author of the article to include these quotes in the article.

    So what was in the article and what was its purpose? Tell us about the latest findings in the field that excite everyone involved in the field. Dark photons, force between dark particles and not a dark particle.., how beautiful. It's been a long time since I read an article that updated me so much and drove me to the wiki. Cheers to Idan!

    So I don't understand how you read such an article and all you take from it is its first paragraph, which ends with a question mark(!), as the purpose of the article.

  169. Yehuda
    Your understanding of the article is the opposite of mine. The article is optimistic and nowhere claims that the Hamel material idea is wrong

    Yehuda - your approach is terribly unscientific. Do you have another hypothesis that solves the problems that the idea of ​​dark matter solves? I don't think so. You offer a formula that fits some of the results, and does not explain other phenomena that dark matter does solve (for example, as I understand it, gravitational decay).

    And as Alessandro said - if your formula contradicts general relativity - then we'll throw that out too?

    Be serious….

  170. to Albenzo
    See the difference between us. I read the article and see in it the desperation of the writer in which he admits that to this day there is no proof of the reality of the dark mass and he hopes that this research will bring some kind of proof.
    You, on the other hand, are already sure in advance that everything is proven, the dark matter is an existing fact, and in fact people like Yehuda are just confusing their heads. So tell me Albanzo, what is the purpose of this article if everything is proven in the dark matter business?

    Another thing I see here is the most basic things in science such as when a scientific idea should be replaced, and if a formula (for example gravitation) that proved itself at a distance of less than a thousand light years (the inner solar system) must also be true at distances of millions of light years. These are very basic things and the great knowledge you flaunt, your tremendous knowledge of physics, for example of large energies, etc., does not belong to the aforementioned basic things and does not add to the understanding of things, and probably even detracts.
    So, with all my appreciation for your knowledge, it is off topic here.

    And when I call the idea of ​​dark matter in the universe a delusional idea, I want those who read my response to understand my opinion on it, to understand that what is created here is not mass or matter. What is created here is the change of the data measured in the field to fit the formula or as you say "the existing scientific idea", a change that should not be made and is against the most basic rule in science that discusses the question: when should a bad scientific idea be thrown away.

    Albanzo, throw away the gravitational idea, it is not suitable for large distances.
    And true, one has to be a scientist with balls to do such an act of canceling gravitation almost completely for the great cosmological distances.

    Finally, unfortunately, I probably didn't respond to you anymore. I'm just tired of your arrogant attitude towards me, your profanity and insulting derogatory names. I am a graduate of Shankar College in industrial management with honors and I ran a business that dealt with plant optimization for over forty years quite successfully. In my humble opinion, this knowledge is sufficient and allowed for the analysis of the dark matter problem. No need for your extensive knowledge.

  171. Yehuda,
    I'm trying to understand... Assuming that you have, in your opinion, an alternative model that explains the phenomena in question, have you tried to publish it in a scientific journal? Have you tried teaming up with a physicist who supports your model and is willing to help you get things published? You've held these views for years, you must have tried to do something about it, right?

  172. Yehuda,

    1. Yes, I am one of the few. You know, me and a few others who are sitting here next to me in the corridor of the high energy physics department. And some physicists from the nearby university. And a few more from the university next to it. And those in the rest of the US, and those in Europe, and Asia, and Israel... you know, such a tiny minority of 99% of physicists. And even the tiny fraction of physicists who think there is no dark matter, like Milgrom, will not agree with you. Because they - unlike you - bothered to learn what it was about. We have examined the evidence, the evidence, the observations, we know the theoretical models. Milgrom understands that no matter what is written in a book for 15-year-old high school students - there are no magic formulas in physics. You can't just decide that instead of the distance to the power of minus two you put minus two and a half. All connections arise from the model, in our case general relativity, and if you want to change the "formula", you have to invent a completely new theory of gravitation. And he's been trying for 30 years to do it, and despite the respect I have for him (full disclosure: I know him personally), his idea just doesn't work. For decades it has either stood still, or been beaten by observations that simply show that it is not true (like the bullet cluster I mentioned earlier, and I saw that Sean Carroll also talks about it in the lecture that Shmulik referred to).

    2. Of course, you have nothing substantive to say, so you turn to all kinds of impressions and formulations of a reporter on the science website, who is not a physicist in the field at all. So from her formulations you get the feeling that she is not sure about the existence of dark matter. what do i care Science is not done according to "what impression does Yehuda Sabdarmish get from the wording of a journalist who tells about science (and does not do it herself)", but according to studies. Evidence, observations, experiments, mathematics. You know, all those things you're afraid or too lazy to try to learn. Of course, it's better to cover your ears and shout loudly, "There is no dark mass!"

    3. And if we have already talked about "dark mass", then I agree with you that names are not what is important. I also hope you don't mind that instead of quantum mechanics we will talk about the "idea of ​​quanta", instead of general relativity we will study "general relativity", and instead of Newtonian mechanics we will study "Isaac's theory of motion". Obviously the names are not important. But when you make up your own language, you just sound like an idiot. And to be precise - what exists is the material. Matter can be massive, and it can also be non-massive. It is a property of matter. The matter is dark, not the mass.

    4. So I'm just talking and I have no content to say, huh? is funny. Have you ever heard of "Hafosel in Momo Fosel"? You don't write anything in your comments, except for calling me "Mr. Scientist", "The Light of Science", "The Scientist of the Scientists" these were of course sarcastic expressions aimed at belittling my scientific ability. I, on the other hand, presented a number of concrete arguments which you avowedly ignore. For example, that you didn't bother to study the evidence and observations for the theory you are dismissing. For example, that your claims are factually incorrect (changing the gravitation "formula" will not solve the problem - on the contrary, it will create many more problems because any phenomenon explained by gravitation will cease to be explained if you change general relativity). For example, your claim that the physicists change the data instead of the theory is simply a complete lie. My response was full of arguments. But since you are too stupid/too cowardly to face them, then you claim that I have not given any substantive argument. Oh well.

    5. Of course you are also a little liar, because I never said that "anyone who thinks otherwise is obviously uninformed, stupid, ignorant, etc., etc., etc." I have explained many times that I greatly respect people's attempts to find alternative teachings and explanations that do not include dark matter. What to do if they fail, while dark matter only accumulates more and more evidence of its existence. And as I've already said - long live the small difference between a physicist who studies the subject and tries to find a real solution to the problem, and some unscrupulous person who has made an internet career out of entering scientific forums and spreading wrong and false information about dark matter. As I told you, I personally know the people who study alternative teachings and I *guarantee* you that if they read your comments they would be at least as shocked as I am, if not more.

    6. Do you think the conclusions from my knowledge are incorrect? How do you know? You have no idea what my knowledge is. At least 5 (I think closer to 10) times I tried to get you to read about the theory you are so opposed to. I gave you links to peer-reviewed articles on the theory. I gave you links to scientific papers that list the positive evidence for the existence of dark matter. I recommended you books that review the subject. Of course you never bothered to study. So if you have no idea what I know about dark matter, how exactly do you decide that my conclusions are wrong?

    You can resent my style. You can cuddle up to Nissim, who for some reason treats you with silk gloves. But the bitter truth is that you are no different from the flood of religious reactionaries (and just paranoid-schizophrenics who think Flash technology came from outer space) that has been dominating the site for the past few months. You all suffer from exactly the same disease - you decided what was right and what was wrong before you bothered to open a book or perform an experiment. You see, what Milgrom does is science. Taking the data, the measurements, the evidence, trying to build mathematical models that will provide predictions, and testing them. What you do is the antithesis of that. So even though you both support an explanation for gravity that doesn't include dark matter, you couldn't be more different. He's a scientist, and you're trying to take us back to the 16th century. good luck with that.

  173. Yehuda, in the matter of terminology you can still give up, although I don't understand why you insist on being wise and using your personal terminology instead of the ones accepted by everyone.

    Regarding the matter itself, it was explained to you that there are observations that give strong confirmation to the dark matter theory, why do you ignore them? Or alternatively, how do you explain them better without the dark matter?

  174. Yehuda, regarding the terminology, you can still give up, although I don't understand why you insist on being wise and not using the terminology accepted by everyone?

    But regarding the matter itself, it was explained to you that there are observations that give strong confirmation to the dark matter theory, why do you ignore them or how do you explain them better without the dark matter?

  175. Yehuda, regarding the terminology you can still give up, although I don't understand why you insist on being wise and not using the terminology accepted by everyone?

    But regarding the matter itself, it was explained to you that there are observations that give strong confirmation to the dark matter theory, why do you ignore them or how do you explain them better without the dark matter?

  176. Lesbri Marnan
    With all the appreciation I have for Albanzo's knowledge (and I have appreciation.... and even a little envy...) I think the conclusions from his knowledge are "a little" wrong and I don't have to agree with them. And what to call the idea of ​​mass / matter / illusory / dark / imaginary / fictional / transparent / it really doesn't matter as long as you understand that it is the biggest scientific disaster of the twentieth century!!!
    Please respond gently, this is science!
    Sabdarmish Yehuda

  177. Yehuda, it's a bit funny that after elbentzo corrects you and explains that you should say "dark matter" and not "dark mass" you insist on continuing to call it "dark mass".

  178. June,
    I suggest you watch the YouTube I attached. He explains in a good way what we discovered empirically (including by Milgrom) and where MOND and TeVes come into the picture

  179. יוני
    Albanzo explained again and again - the dark matter is not an invented "solution" to the problems of gravity, rather, there are many additional observations, which confirm that the dark matter does exist.

    Changing the gravity formula does not answer the other types of observations, and it explains nothing.

  180. No need to insult..
    Yehuda thinks like the alternative theory developed by Prof. Motty Milgrom from the Weizmann Institute...
    http://www.themarker.com/magazine/1.2208993
    The entire generation of dark matter - stems from the difference between the two methods of calculating the amount of mass... the fact that everyone went for the first solution - does not justify that it is actually the right one...

  181. More about MOND and TeVes
    Here is a talk by Shane Carroll where he talks about ideas on how to move beyond general relativity. At about minute 31:20 he starts talking about MOND and Teves and at about minute 40 he gets to the point of why these two ideas fail to get rid of dark matter.

    http://youtu.be/SwyTaSt0XxE

  182. Yehuda
    How is this disprovable? All you do is change the formula again.
    What I'm saying is that you set out to explain why there are these strange phenomena.. What you're doing, in my opinion, is equivalent to Ptolemy's epicycles and diferents. Add enough - it will work, and if it doesn't work then we'll add another one.

    That still doesn't explain anything...

  183. Yehuda

    You are the only person writing here,
    which casts doubt on "scientific" determinations.

    For some reason, they don't attack you, like they attack me...

    Maybe because you are very, very careful not to step on warts...

  184. to Albenzo
    One thing can be said about you that you are one of the few who believe in the reality of the dark mass without any doubt. Everyone else, including the authors of the article in question, have enough. The author of the article calls the reality of the dark mass "a bleak scenario." He begins the article with the words:
    "There is something somewhere in the universe. We can't see it, we can't touch it and we only know it's there because of the gravitational pull it exerts on the objects in the universe. For decades, the story of dark matter has been discovery after discovery about what this mysterious substance is, a gradual filtering of possibilities that has made physicists feel increasingly nervous. What will happen when the last candidate is deleted from the list? Will our fate be sealed, never to get a glimpse of the nature of the matter that contributes about 25% of the total mass in the universe?" End quote.
    It seems to me that the author of the article also understands that eighty years of searching is more than enough and he ends the article with the words "In my opinion, it's now or never."
    But to you, Mr. Albanzo scientist, the light of science, the scientist of scientists, there is no insufficiency, the dark mass exists, without a doubt because of all kinds of bombastic words you put in your response. And anyone who thinks otherwise is obviously uninformed, stupid, ignorant, etc., etc., etc. Forgive me, but you are wasted. Start thinking outside the box and you might come to my conclusions.
    Unfortunately, apart from belittling and slandering my response, you didn't say anything about the doubts that have existed on the topic of dark mass for 80 years. You have no doubts. Too bad!

    for miracles
    Why do you state that the gravitation formula cannot be disproved?, for your information it was disproved by testing it in spiral galaxies and galaxy clusters. Just because someone decided to change the data with the illusory mass doesn't mean it hasn't been disproven.
    Just, that's my opinion!
    Please respond gently.
    Sabdarmish Yehuda

  185. Yehuda
    I'll tell you how I see it. You look at the measurements, fit a formula to them, and go home.

    What is this and science? There is no science here - it doesn't explain anything, it doesn't predict anything, and it can't be disproved.

  186. When will the scientists wake up and do what you say? When the universities will stop teaching physics. When scientists decide that it is not suitable for them to study the material they are researching, that it is enough to read an enrichment book for high school students and base all our knowledge on it. In short, when all the physicists start being really, really stupid.

    As usual, Yehuda writes a pile of nonsense. But there is one nonsense that must be addressed privately, and there is one thing he writes that is actually not nonsense.

    The special nonsense is that "it is enough to look for the illusory mass". From this nonsensical statement, in particular from the words "illusory mass", it follows that it does not exist. So how to tell you this, we found her already. Dark matter (it's enough to say "dark mass", no one except Yehuda Sabdarmish calls dark matter that way) has already been found. There are sightings of it in all kinds of places in space. What is still missing is a small piece of it, here, on the Earth, which allows us to study its microscopic properties (such as its composition, charges under different forces, etc.). But dark matter in large clusters has been observed for years. A nice example can be seen, for example, in the bullet cluster.

    The correct thing Yehuda said is that when a mathematical model does not fit the measurements, the model should be thrown away (please stop using the word "formula", for hundreds of years there have been no "formulas" in physics. There are mathematical models from which certain relationships arise, but it is impossible to change the relationships without changing the the model). And really - there used to be a model that had many names, for example FRW, and they thought it was excellent. Then they continued a little further, and found that it did not fit the measurements. The model - which had no dark matter or dark energy - simply did not agree with reality. So they threw it away. Looked, looked, and found a new model. The new model matched the measurements fantastically. The new model had predictions that were adjusted in the laboratory. The new model had positive evidence for its correctness. The model was called LCDM, or in Hebrew - "Lamda CDM". Lamda is the Greek letter that represents dark energy, and CDM stands for Cold Dark Matter. Answer, cold dark matter. But Yehuda wants us to throw away this model. Why? Because he doesn't understand it... In fact, he is so afraid of it that every time he is brought a link to a review of the model, including the evidence of its correctness (with measurements - it is not something theoretical but verified in the laboratory), he refuses to read it with some flimsy excuse ("It's in English ", "It's long", "I don't have time"...).

    Yehuda, do you want them to respond to you gently? Stop writing for years and years the same dumb response that stems from ignorance (you talk about things you have no idea about and you avowedly refuse to learn them) and slander people who are 10 times smarter than you, 100 times more honest than you, and 1000 times more diligent than you who are actually trying to promote our understanding of the universe around us and not take it back to the Middle Ages.

  187. In the first lesson in high school in physics, the teacher said that if the results obtained from a formula do not match the data measured in the field, then who should be thrown away, the results measured in the field or the formula? It is clear to everyone that one should do a simple thing and that is to throw away the formula and look for another. It's clear to everyone, but sometimes you need a little courage. When will brave scientists stand up and decide that the gravitation formula has exhausted itself and after 350 years since Newton's days it must be replaced by another one. The movement in the galaxies must be explained without the holy gravitation!, there is no sense in sticking to it, and changing the data with illusory substances such as dark matter and energy whose entire function is to preserve the gravitation formula in its form. Eighty years have passed since Fritz Zviki came up with the idea of ​​dark mass to preserve the Newton-Einstein gravitation formula even at the great distances of galaxies and galaxy clusters. It is enough to look for the illusory mass. It's time to change.
    Please respond gently!
    Sabdarmish Yehuda

Leave a Reply

Email will not be published. Required fields are marked *

This site uses Akismat to prevent spam messages. Click here to learn how your response data is processed.